index [iasparliament.s3.ap-south-1.amazonaws.com]

69

Upload: others

Post on 02-Jan-2022

0 views

Category:

Documents


0 download

TRANSCRIPT

Page 1: INDEX [iasparliament.s3.ap-south-1.amazonaws.com]
Page 2: INDEX [iasparliament.s3.ap-south-1.amazonaws.com]

3

www.shankariasacademy.com | www.iasparliament.com

INDEX

01-09-2021 ...................................................................................................................................... 4

02-09-2021 ..................................................................................................................................... 8

03-09-2021 .................................................................................................................................... 13

04-09-2021 .................................................................................................................................... 15

06-09-2021 .................................................................................................................................... 17

07-09-2021 ................................................................................................................................... 22

08-09-2021 ................................................................................................................................... 27

09-09-2021 .................................................................................................................................... 31

11-09-2021 .................................................................................................................................... 33

13-09-2021 .................................................................................................................................... 36

14-09-2021 .................................................................................................................................... 37

15-09-2021 .................................................................................................................................... 40

16-09-2021 .................................................................................................................................... 42

17-09-2021 .................................................................................................................................... 44

18-09-2021 .................................................................................................................................... 46

21-09-2021 .................................................................................................................................... 48

22-09-2021 ................................................................................................................................... 50

23-09-2021 ................................................................................................................................... 52

24-09-2021 ................................................................................................................................... 55

25-09-2021 ..................................................................................................................................... 57

27-09-2021 .................................................................................................................................... 60

28-09-2021 ................................................................................................................................... 62

29-09-2021 ................................................................................................................................... 64

30-09-2021 ................................................................................................................................... 67

Page 3: INDEX [iasparliament.s3.ap-south-1.amazonaws.com]

4

www.shankariasacademy.com | www.iasparliament.com

01-09-2021

1) China has recently inaugurated a trade corridor linking the Indian Ocean with south-western China. With reference to the trade corridor, consider the following statements:

1. The transport corridor comprises three modes of transport viz, sea-road-rail link.

2. It was developed as a part of China Pakistan Economic Corridor (CPEC).

3. The passage connects the logistics lines of Pakistan, Myanmar and China.

Which of the statement(s) given above is/are correct?

a. 1 only

b. 3 only

c. 2 and 3 only

d. 1, 2 and 3

2) With reference to UNSC Resolution 2593, adopted recently, consider the following statements:

1. The resolution called upon host States to promote accountability for the killing and all acts of violence against the UN peacekeeping personnel.

2. It specifically mentions individuals designated by the UNSC resolution 1267, which includes the Lashkar-e-Taiba and the Jaish-e-Mohammad.

3. All the 15 members of the Council voted unanimously in favour of the resolution.

Which of the statement(s) given above is/are correct?

a. 2 only

b. 1 and 3 only

c. 2 and 3 only

d. 1, 2 and 3

3) Consider the following statements:

1. The river flows in both India and Bangladesh and act as a natural boundary between Jaintia and Khasi Hills.

2. Over this river hangs a single span suspension bridge called Dawki Bridge.

3. The river is considered as the cleanest river in India and in some parts is as transparent as crystal and you can actually see the river bed.

4. The river passes through the Asia‘s Cleanest Village, Mawlynnong.

Identify the river that correctly matches with the above description:

a. Kynshi River

b. Umngot River

c. Jinjiram River

d. Simsang River

4) Consider the following statements with respect to Self-Reliant India (SRI) Fund

1. It is an Alternative Investment Fund to provide growth capital for Micro, Small and Medium Enterprises (MSMEs).

2. The fund will have a Mother Fund/Daughter Fund structure with a corpus of Rs.10,000 crore as Mother Fund.

3. Govt. of India will be the sole anchor investor of the fund.

Which of the statement(s) given above is/are correct?

a. 1 and 2 only

b. 2 and 3 only

c. 2 and 3 only

d. 1, 2 and 3

5) Which of the following passes is/are located in Ladakh?

1. Kela Pass

2. Umling La

3. Khardung La

Select the correct answer using the codes given below:

a. 3 only

b. 1 and 3 only

c. 2 and 3 only

d. 1, 2 and 3

Page 4: INDEX [iasparliament.s3.ap-south-1.amazonaws.com]

5

www.shankariasacademy.com | www.iasparliament.com

6) Consider the following statements about The Factoring Regulation (Amendment) Act 2021

1. It allowed the participation of NBFCs in the TReDS (Trade Receivables Discounting System) platform.

2. For NBFCs, the income from the factoring business should be more than 50% of the gross assets/net income.

Which of the statements given above is/are correct?

a. 1 only

b. 2 only

c. Both 1 and 2

d. Neither 1 nor 2

7) Which of the following are the impacts of using Antibiotics in cows?

1. Decrease in ability of the Soil to store carbon

2. Change in Soil‘s bacterial community structure

3. More carbon in the atmosphere in the form of CO2

Choose the correct option:

a. 2 only

b. 1 and 3 only

c. 2 and 3 only

d. 1, 2 and 3

8) Which of the following countries has recently declared Food Emergency due to drain of Forex to finance imports?

a. Morocco

b. Sri Lanka

c. Afghanistan

d. Sudan

9) Consider the following statements about Attingal Outbreak

1. It is one of the earliest organised rebellions against the East India Company that erupted way back in 172

2. Maharaja of Mysore rose against the corrupted East India Company officers involved in the monopoly of spices.

3. The construction of Anjengo fort by the EIC sparked rivalry between native ruler and EIC.

Which of the statements given above are correct?

a. 1 and 2 only

b. 1 and 3 only

c. 2 and 3 only

d. 1, 2 and 3

10) Consider the following statements about Cyanobacteria

1. Cyanobacteria are prokaryotes belong to Monera kingdom and often called as Blue-Green Algae.

2. They are mainly aquatic that phosynthesize their own food.

3. It is capable of producing toxins that can be ingested by livestocks and enter the food chain.

Which of the statements given above is/are correct?

a. 1 only

b. 2 only

c. 2 and 3 only

d. 1, 2 and 3

Answers

1. a

The first shipments on a newly-launched railway line from the Myanmar border to the key commercial hub of Chengdu in western China were delivered last week.

Transport Corridor

It involves a sea-road-rail link.

This passage connects the logistics lines of Singapore, Myanmar and China, and is currently the most convenient land and sea channel linking the Indian Ocean with southwest China.

Goods from Singapore reached Yangon Port, arriving by ship through the Andaman Sea of

Page 5: INDEX [iasparliament.s3.ap-south-1.amazonaws.com]

6

www.shankariasacademy.com | www.iasparliament.com

the northeastern Indian Ocean, and were then transported by road to Lincang on the Chinese side of the Myanmar-China border in Yunnan province.

The new railway line that runs from the border town of Lincang to Chengdu, a key trade hub in western China, completes the corridor.

The one-way journey saves 20 to 22 days.

Other Plans of China

Kyaukphyu Port - China also has plans to develop another port in Kyaukphyu in the Rakhine state of Myanmar including a proposed railway line from Yunnan directly to the port, but the progress there has been stalled by unrest in Myanmar.

Gwadar Port - Chinese planners have also looked at the Gwadar port in Pakistan as another key outlet to the Indian Ocean that will bypass the Malacca Straits.

Gwadar is being developed as part of the China Pakistan Economic Corridor (CPEC) to the far western Xinjiang region, but has been slow to take off amid concerns over security.

2. a

UNSC Resolution 2593

The resolution demanded that Afghan territory not be used to threaten or attack any country and reiterated the importance of combating terrorism in Afghanistan.

The resolution adopted by a vote of 13 in favour with two abstentions (Russian Federation and China).

The resolution also urged Talibans‘ to assist the safe evacuations of all Afghan nationals wishing to leave the country.

It specifically mentions individuals designated by the UNSC resolution 1267, which includes the Lashkar-e-Taiba (LeT) and the Jaish-e-Mohammad (JeM).

3. b

The Meghalaya government has recently scrapped an agreement with private power developers to execute the proposed 210 MW Umngot hydroelectric project following protests.

Umngot River

It flows both in India & Bangladesh.

It separates East Khasi Hills District from Jaintia Hills District by creating a boundary in between the two.

It is the natural boundary between Ri Pnar (of Jaintia Hills) with Hima Khyrim (of Khasi Hills).

Over the river hangs a single span suspension bridge called Dawki Bridge.

Umngot River is considered as the cleanest river in India and in some parts is as transparent as crystal and you can actually see the river bed.

Umngot River is also called by the name Dawki River, which has greenish-bluish color transparent water.

The Umngot River passes through the village of Mawlynnong in Meghalaya, close to India‘s border with Bangladesh, which is touted as ―Asia‘s Cleanest Village".

4. d

Self Reliant India (SRI) Fund

It is an Alternative Investment Fund (AIF), for providing growth capital to the Micro, Small and Medium Enterprises (MSMEs), through equity/quasi-equity/equity like structured instruments.

This will facilitate them to move towards listing on the Stock Exchanges and in becoming National and International Champions by growing beyond the bracket of MSME.

The Fund is to be anchored by NSIC Venture Capital Fund Limited (NVCFL).

NVCFL is a wholly owned subsidiary of The National Small Industries Corporation.

Structure of the fund - SRI Fund will have a Mother Fund / Daughter Fund structure with a corpus of INR 10,000 crore as Mother Fund.

Govt. of India will be the sole anchor investor and provide an initial budgetary support of Rs. 10,000 crore to the Mother Fund in phased manner.

Mother Fund will provide funds only to the Daughter Funds for onward investment as

Page 6: INDEX [iasparliament.s3.ap-south-1.amazonaws.com]

7

www.shankariasacademy.com | www.iasparliament.com

growth capital, while the investment in MSMEs will be done by the Daughter Funds.

Both the Mother and Daughter Funds will be duly registered as Alternate Investment Funds with SEBI.

Fund Life - The SRI Fund will have a life of 15 years and it will be a revolving fund.

5. d

6. a

The Factoring Regulation (Amendment) Act 2021

Factoring business is a business where an entity acquires the receivables of another entity for an amount.

Factor can be a bank, a registered non-banking financial company or any company registered under the Companies Act.

Receivables are the total amount that is owed or yet to be paid by the customers (referred as the debtors) to the assignor for the use of any goods, services or facility.

The Act allowed participation of non-banking finance companies (NBFCs) in the TReDS platform.

It will bring in over 9,000 NBFCs into the factoring business against the current participation of just 7 NBFCs.

The act removes the threshold of NBFCs in which 50% of the gross assets/net income from factoring business.

It also amended the definitions of "receivables", "assignment", and "factoring business" to bring them at par with international definitions.

TReDS is an electronic platform for facilitating financing of trade receivables of Micro, Small and Medium Enterprises.

7. d

Effects of Antibiotics in soil

Carbon in soils exposed to manure from cows administered with antibiotics, travelled into the above-ground plant material, to the roots of the plants, into the soil and respired back out as CO2 much faster than any of the others.

This means that when antibiotics are used, less carbon is stored in the soil and more is lost to the atmosphere as CO2

It can change soil microbiome and decrease their ability to store carbon

It also affects soil respiration and elemental cycling

It also caused changes in the soil's fungal and bacterial ratios as well as altered the bacterial community structure.

8. b

Sri Lanka has recently declared a state of emergency over food shortages as private banks run out of foreign exchange to finance imports.

The emergency regulation allows government officials to seize food stocks held by traders and arrest people who hoard essential food.

Sri Lanka‘s foreign reserves fell to $2.8 billion at the end of July 2021, from $7.5 billion in November 2019.

Tourism was the only vital source of foreign exchange earnings, but that too suffered due to the coronavirus pandemic.

It has more foreign debt payments of $1.5 billion each due in the next 12 months. It has already paid $1.3 billion so far.

9. b

Attingal Outbreak

The uprising happened in 1721 in the State of Travancore.

The East India Company came to Attingal calling for the pepper from Nedumangad, one of the most sought-after spices in Europe.

EIC got the permission to build a fort called Anjengo in 1694 from the queen.

They also got exclusive rights to buy pepper at a much cheaper rate than what the Dutch were offering.

Following the civilian unrest, the queen ordered the British to stop the construction of the fort but they refused. It was the first instance of rivalry between Native ruler and EIC.

Page 7: INDEX [iasparliament.s3.ap-south-1.amazonaws.com]

8

www.shankariasacademy.com | www.iasparliament.com

After crushing the native troops, the fort became full settlement of the English.

William Gyfford was appointed as the new officer of the settlement to rein in corruption as some officers were indulging in private trade.

By the 1720s, the British dictated the terms of merchandise in Attingal.

The natives were fed up with the unethical trade practices of the British and mocking of their religious practices by British.

10. d

Cyanobacteria

Scientists have identified a set of genes in cyanobacteria that can help boost energy yield from photosynthesis, and also pave way for a sustainable biotech production.

Cyanobacteria are aquatic and photosynthetic and are often called blue-green algae.

They have the distinction of being the oldest known fossils, more than 3.5 billion years old.

The cyanobacteria have also been tremendously important in shaping the course of evolution and ecological change throughout earth's history.

The oxygen atmosphere that we depend on was generated by numerous cyanobacteria during the Archaean and Proterozoic Eras.

The other great contribution of the cyanobacteria is the origin of plants. The chloroplast with which plants make food for themselves is actually a cyanobacterium living within the plant's cells.

Dead and dying cyanobacteria, which contain preformed toxins saccumulate on the surface of bodies of water and are ingested by livestock.

The bacterial species share with plants an enzyme called rubisco, which plays a key role in photosynthesis.

02-09-2021

1) Consider the following statements with respect to Black-necked Crane

1. The species is found in India, China and Bhutan.

2. It has been classified as Vulnerable under the IUCN Red list of Threatened species.

3. The major threat to the successful breeding of black-necked crane is the damage to the eggs and chicks, caused by feral dogs.

Which of the statements given above are correct?

a. 1 and 2 only

b. 1 and 3 only

c. 2 and 3 only

d. 1, 2 and 3

2) Consider the following statements with respect to Snow Leopard

1. The range of the species extends from the Himalaya in the south to the mountains of southern Siberia in the north.

2. It has been listed as Vulnerable under the IUCN Red list of threatened species.

Which of the statement(s) given above is/are correct?

a. 1 only

b. 2 only

c. Both 1 and 2

d. Neither 1 nor 2

3) Consider the following statements with respect to ZAPAD 2021

1. It is one of the theatre level exercises of Russian armed forces that focus primarily on operations against terrorists.

2. India is one among the 9 participating countries that are invited for the exercise.

3. China, Pakistan is taking part as observers in this multinational exercise.

Which of the statement(s) given above is/are correct?

a. 1 and 2 only

b. 1 and 3 only

c. 2 and 3 only

d. 1, 2 and 3

Page 8: INDEX [iasparliament.s3.ap-south-1.amazonaws.com]

9

www.shankariasacademy.com | www.iasparliament.com

4) With respect to a recent report, Animal Discovery, 2020, consider the following statements:

1. Animal Discoveries are the only authentic source of faunal discoveries of India that has been published since 2009 every year.

2. It was prepared by the Zoological Survey of India (ZSI).

3. According to the report, around 239% of India‘s geographical area is under forest and tree cover.

Which of the statement(s) given above is/are correct?

a. 1 and 2 only

b. 1 and 3 only

c. 2 and 3 only

d. 1, 2 and 3

5) Panjshir Valley, often seen in the news recently, is located in?

a. Pakistan

b. Mongolia

c. Tajikistan

d. Afghanistan

6) Consider the following statements about Direct Seeding of Rice

1. It involves no nursery preparation of seeds and requires less water than conventional transplanting method.

2. It uses chemical herbicides.

3. The seed requirement is very high.

Which of the statements given above is/are correct?

a. 1 only

b. 1 and 3 only

c. 2 and 3 only

d. 1, 2 and 3

7) Consider the following statements about State of the World‘s Trees

1. According to the report, Climate change is the leading factor for the increase in threatened tree species.

2. Madagascar is one of the countries with the highest number of threatened trees.

Which of the statements given above is/are correct?

a. 1 only

b. 2 only

c. Both 1 and 2

d. Neither 1 nor 2

8) Consider the following statements about Hydro-meteorological calamities

1. Madhya Pradesh and West Bengal lost the most number of lives due to hydro-meteorological calamities in the last 4 years.

2. Madhya Pradesh and Karnataka lost the maximum crop area due to the calamities.

3. Kerala suffered the maximum destruction of homes.

Which of the statements given above are correct?

a. 1 and 2 only

b. 1 and 3 only

c. 2 and 3 only

d. 1, 2 and 3

9) Consider the following statements about new variant of SARS-COV

1. The latest variant B.1.621 has been given label ‗Mu‘ and classified as ‗Variant of Interest‘.

2. It was first identified in Colombia in 2021.

Which of the statements given above is/are correct?

a. 1 only

b. 2 only

c. Both 1 and 2

d. Neither 1 nor 2

10) Consider the following statements about Allocation of Special Drawing Rights (SDR) to India

Page 9: INDEX [iasparliament.s3.ap-south-1.amazonaws.com]

10

www.shankariasacademy.com | www.iasparliament.com

1. India has recently been allocated 25% of overall SDRs general allocation made to the IMF‘s member countries.

2. SDR allocation is made to IMF countries to reduce their reliance on more expensive domestic or external debt for building reserves.

Which of the statements given above is/are not correct?

a. 1 only

b. 2 only

c. Both 1 and 2

d. Neither 1 nor 2

Answers

1. b

Black-necked crane was recently adopted as State bird by the Union Territory of Ladakh.

Black Necked Crane

Distribution - This species is found in India, China and Bhutan and breeds in high altitude wetlands in the Tibetan plateau at elevations of 2950-4900 m above mean sea level.

Characteristics - Both the sexes are almost of the same size but male is slightly bigger than female.

Conservation Status - It has been listed in Schedule I of Wildlife (Protection) Act 1972 and as Near Threatened on the IUCN Red List.

Major Threats - The damage to the eggs and chicks, caused by feral dogs is one of the major threat to the species.

These dogs are owned both by armed forces as well as by the local nomads.

Another threat to the bird is the loss of habitat.

2. c

Snow Leopard was recently adopted as State animal by the Union Territory of Ladakh.

Snow Leopard

The Snow Leopard is also known as Ghost of the mountains.

It acts as an indicator of the health of the mountain ecosystem in which they live, due to their position as the top predator in the food web.

The Snow Leopard lives at high altitudes in the steep mountains of Central and Southern Asia, and in an extremely cold climate.

They inhabit the higher Himalayan and trans-Himalayan landscape in the states/union territories of Jammu and Kashmir, Himachal Pradesh, Uttarakhand, Sikkim, and Arunachal Pradesh.

India is a unique country to have a good presence of 5 big cats, including Snow Leopard.

The other 4 are: Lion, Tiger, Common Leopard, and Clouded Leopard.

Snow Leopard capital of the world is Hemis National Park, Ladakh.

The animal is protected under the following conventions:

1. IUCN Red List- Vulnerable

2. Convention on International Trade in Endangered Species (CITES)- Appendix I

3. Convention on Migratory Species (CMS)- Appendix I

Page 10: INDEX [iasparliament.s3.ap-south-1.amazonaws.com]

11

www.shankariasacademy.com | www.iasparliament.com

4. Appendix I includes species threatened with extinction.

5. Indian Wildlife (Protection) Act 1972- Schedule I

3. d

A contingent of 200 Army personnel of Indian Army will participate in the multinational Exercise ZAPAD 2021 being held at Nizhniy, Russia.

ZAPAD 2021

It is one of the theatre level exercises of Russian armed forces that focus primarily on operations against terrorists.

Of the 17 countries invited for the exercise, there are nine participating countries and eight Observers.

The nine Participating countries includes Mongolia, Armenia, Kazakhstan, Tajikistan, Kyrgyzstan, Serbia, Russia, India and Belarus.

The other eight Observer countries are Pakistan, China, Vietnam, Malaysia, Bangladesh, Myanmar, Uzbekistan and Sri Lanka.

4. d

Animal Discovery, 2020

Recently, Animal Discovery 2020 was prepared by the Zoological Survey of India (ZSI) and released by the Environment Ministry.

It reveals that India has added 557 new species to its fauna. Now, there are 1,02,718 faunal species in India.

It shows that India is positioned 8th in mega biodiversity countries in the world with 0.46 BioD index. Around 23.39% of India‘s geographical area under forest and tree cover.

BioD index is calculated by its percentage of species in each group relative to the total global number of species in each group.

Animal Discoveries

It is the only authentic source of faunal discoveries of India.

It is published by the ZSI since 2009 every year.

It is a document on new species and new records of fauna.

Zoological Survey of India

Set up by British zoologist Thomas Nelson Annandale in 1916, the ZSI is India‘s apex organization on animal taxonomy.

It is a subordinate organization of the Environment Ministry.

Headquartered in Kolkata, the ZSI BSI has 16 regional circles at different regions of the country.

ZSI promotes survey, exploration and research leading to the advancement of knowledge on many aspects of exceptionally rich faunal diversity of India.

It has been declared as designated repository for National Zoological Collection under the National Biodiversity Act, 2002.

5. d

The Taliban had recently called on fighters in the Panjshir Valley to lay down their arms.

The Panjshir Valley is the centre of Afghanistan‘s most important pocket of armed anti-Taliban forces.

The valley stood strong against the Taliban rule from 1996-2001 before the US-led foreign troops invaded Afghanistan.

Panjshir Valley

Located 150 km north of Kabul, Afghanistan, the Valley is near the Hindu Kush mountain range.

It‘s divided by the Panjshir River and ringed by the Panjshir Mountains in the north and the Kuhestan mountains in the south.

The mountain tops are covered by snow throughout the year.

This difficult terrain makes the Valley a nightmare for invaders.

6. d

Direct Seeding of Rice (DSR)

In the conventional transplantation, farmers prepare nursery seedbed to be transplanted but in DSR, the paddy seeds are directly drilled into the field by a tractor-powered machine.

Page 11: INDEX [iasparliament.s3.ap-south-1.amazonaws.com]

12

www.shankariasacademy.com | www.iasparliament.com

Conventional transplantation required the paddy field to be flooded to prevent the growth of herbicides.

In DSR, chemical herbicides are used in two stages – Pre-emergent and post-emergent and it leads to water savings.

Labour - About three labourers are required to transplant one acre of paddy in a single day.

The seed requirement for DSR is also higher, at 8-10 kg/acre, compared to 4-5 kg in transplanting.

Laser land levelling, which costs Rs 1,000/acre, is compulsory in DSR. This is not so in transplanting.

7. b

State of the World’s Trees

The report is released by London-based Botanic Gardens Conservation International.

Report Highlights -

India is home to 650 endemic tree species that are not found anywhere else.

Threatened Species -

o The Palearctic and Nearctic (North America) realms mostly have tree species that are not threatened.

o Indo-Malaya, Oceania, the Afrotropics (Africa south of the Sahara, including Madagascar) has the highest proportion of threatened tree species.

Number of Tree Species -

o The Neotropics (Central and South America) have the largest number of tree species followed by Indo-Malaya, Afrotropics.

o The Nearctic and Oceania have the lowest number of tree species.

o Brazil, China, Colombia and Indonesia had a large number of tree species as well as a large number of threatened species.

o Madagascar is one of the countries with the highest number of threatened trees.

Major Threats -

o Agriculture (29%), logging (27%), livestock farming (14%), Residential and commercial development, fire and fire suppression (13%), energy production and mining (9%), wood and pulp plantations (6%), invasive and other problematic species (5%) and climate change (4%).

8. d

Hydro meteorological calamities

It include floods (flash floods, river floods, coastal floods, urban floods etc.), droughts (meteorological, hydrological agricultural), cyclones, avalanches, heat and cold waves etc.

According to the data presented by the Ministry of Home Affairs in the Parliament,

Madhya Pradesh lost the highest number of its people (970) in four years followed by West Bengal (964), Maharashtra (875), Kerala (764) and Himachal Pradesh (594).

Madhya Pradesh and Karnataka lost the maximum crop area and Kerala suffered the maximum destruction of homes.

9. c

‘Mu’ Variant

Page 12: INDEX [iasparliament.s3.ap-south-1.amazonaws.com]

13

www.shankariasacademy.com | www.iasparliament.com

The latest variant of interest, B.1.621, has been given the WHO label ‗Mu‘ and classified as ‗variant of interest‘.

This includes the descendent Pango lineage B.1.621.1.

It was first identified in Colombia in January 2021.

There have been sporadic reports of the variant from South America and Europe.

The Mu variant has a constellation of mutations that indicate potential properties of immune escape.

10. a

Special Drawing Rights (SDR) by IMF

The SDR is based on a basket of international currencies comprising the US dollar, Japanese yen, euro, pound sterling and Chinese Renminbi.

It is not a currency, nor a claim on the IMF, but is potentially a claim on freely usable currencies of IMF members.

IMF has made an allocation of SDR 12.57 billion (equivalent to around $17.86 billion at the latest exchange rate) to India.

It is about 2.75% of overall SDRs allocated to member countries.

IMF makes the general SDR allocation to its members in proportion to their existing quotas in the IMF.

It is one of the components of the foreign exchange reserves of a country.

03-09-2021

1) Consider the following statements with respect to Anaimalai Flying Frog

1. It is endemic to the southern part of Eastern Ghats, Tamil Nadu.

2. The female creates foam nests on leaves, into which the eggs are laid and the male fertilises them.

3. It was listed as Critically Endangered under the IUCN Red list of threatened species.

Which of the statement(s) given above is/are correct?

a. 1 and 2 only

b. 1 and 3 only

c. 2 and 3 only

d. 1, 2 and 3

2) Ashok Kumar Tandon Committee, sometimes seen in the news recently, was constituted for which of the following purposes?

a. To probe Pegasus snooping issue

b. To reform Journalists Welfare Schemes

c. To suggest measures for Sustainable Fisheries Management

d. To study the impact of traditional health practices in rural villages

3) Consider the following statements with respect to President‘s Colour

1. It is the highest honour bestowed on a military unit in recognition of its exceptional service to the nation.

2. The Indian Army was the first amongst the Indian Armed Forces to be awarded the President‘s Colour in 1951.

Which of the statement(s) given above is/are correct?

a. 1 only

b. 2 only

c. Both 1 and 2

d. Neither 1 nor 2

4) With respect to 1988 sanctions committee, sometimes seen in the news recently, consider the following statements:

1. The committee oversees the sanctions measures imposed by the UN Security Council (UNSC) on Al-Qaeda.

2. India will chair the 1988 sanctions committee for the period from 2021-22.

Which of the statement(s) given above is/are correct?

a. 1 only

b. 2 only

c. Both 1 and 2

Page 13: INDEX [iasparliament.s3.ap-south-1.amazonaws.com]

14

www.shankariasacademy.com | www.iasparliament.com

d. Neither 1 nor 2

5) With respect to Green List of Protected and Conserved Areas (GLPCA), consider the following statements:

1. It is the first global standard of best practice for area-based conservation.

2. It is being developed by International Union for Conservation of Nature (IUCN) and partners.

Which of the statement(s) given above is/are correct?

a. 1 only

b. 2 only

c. Both 1 and 2

d. Neither 1 nor 2

Answers

1. c

Anaimalai Flying Frog

It is also known as the False Malabar Gliding Frog.

It is endemic to the southern part of the Western Ghats.

This species is usually larger than bush frogs.

Mating takes place usually between June and October, during the rainy season.

The female creates foam nests on leaves, into which the eggs are laid and the male fertilises them.

The outer layer of foam protects the eggs from bacteria, predators and weather changes.

When the eggs hatch, the nest disintegrates and tadpoles drop into the water body below.

2. b

Ashok Kumar Tandon Committee

Ministry of Information and Broadcasting has constituted a ten member Committee headed by Shri Ashok Kumar Tandon, renowned Journalist and Member, Prasar Bharati.

The committee was tasked to take a look at the existing guidelines of Journalists Welfare Scheme of the Ministry of Information and Broadcasting.

The Journalists Welfare Scheme which has been in existence for a many years needs a relook from a futuristic perspective and broad basing the coverage for the benefit of the Journalists of this country.

With the enactment of Occupational, Safety, Health and Working Condition Code 2020, the definition of the Working Journalists has been broadened to include within its fold those working in both traditional and digital media.

Further, it was also considered necessary to look at the possible parity between accredited and non-accredited journalists from the perspective of welfare and availing of benefits under the scheme.

3. a

The President of India will award the President‘s Colour to Indian Naval Aviation (INA) at the ceremonial parade to be held at INS Hansa, Goa on 06 Sep 21.

The Indian Navy was the first amongst the armed forces to be awarded the President‘s Colour on May 27, 1951 by then President Rajendra Prasad,

President’s Colour

Since ancient times, the Monarchs and Generals used to fly their flags standards to visually indicate their location to their dispersed forces.

Page 14: INDEX [iasparliament.s3.ap-south-1.amazonaws.com]

15

www.shankariasacademy.com | www.iasparliament.com

In India, this flag was called the "dhwaj".

In battle these flags served as a central point of reference.

If the flag ceased to be visible, it indicated that the percentage had either been defeated, or had fled.

The flag was, therefore, a rallying point, well defended in battle.

In peacetime, it was a focus for pageantry because it was a personal symbol of the Supreme Commander.

4. b

India will chair the crucial Taliban and Libya sanctions committees and the Counter-Terrorism Committee of the UN Security Council (UNSC) during its tenure as non-permanent member of the powerful 15-nation UN body. (2021-22).

1988 Sanctions Committee

It was formed through resolution number 1988 in 2011 by splitting the 1267 sanctions regime on al-Qaeda.

Resolution 1988 dealt with sanctions relating to the Taliban, while Resolution 1989 addressed sanctions on Al-Qaeda.

Until the passing of both the resolutions, sanctions on the Taliban and Al-Qaeda had been handled by the same committee.

The committee oversees the sanctions measures imposed by the Security Council on Talibans.

Individuals and entities are designated on the 1988 Sanctions List as individuals, groups, undertakings and entities associated with the Taliban in constituting a threat to the peace, stability and security of Afghanistan.

The listed entities and individuals are subject to an assets freeze, travel ban and arms embargo.

5. c

Green List of Protected and Conserved Areas (GLPCA)

It is a new global quality standard for protected areas being developed by IUCN and partners.

The intended aims of the IUCN GLPCA are:

1. To recognize and reward effective management and equitable governance of protected areas

2. To assist participating countries to achieve quality in the implementation of their national protected area systems, in part towards meeting and reporting on CBD Aichi Target commitments.

In order to develop the IUCN GLPCA, the IUCN Green List Initiative is under way.

Through this initiative, IUCN and collaborating partners (which include the CA|TS scheme) will develop the global standards and matching guidance that can be used to assess, evaluate and recognize quality in protected areas.

Participating countries will be able to apply local context and criteria for the IUCN Green List process, but the benchmark will be a credible set of IUCN Green List global standards.

04-09-2021

1) Consider the following statements with respect to Climate Adaptive Planning for Resilience and Sustainability (CAP-RES) Programme

1. The mission focuses on five broad thematic areas to address the knowledge gap across the specified themes of resilience and sustainability.

2. The mission was supported by the Department of Science & Technology under the National Knowledge Mission on Climate Change.

3. The mission would be implemented by the National Institute of Disaster Management which works under the Ministry of Home Affairs.

Which of the statement(s) given above is/are correct?

a. 1 and 2 only

b. 1 and 3 only

c. 2 and 3 only

d. 1, 2 and 3

Page 15: INDEX [iasparliament.s3.ap-south-1.amazonaws.com]

16

www.shankariasacademy.com | www.iasparliament.com

2) Consider the following statements with respect to Bio-Methanation

1. It is a process by which organic material is microbiologically converted under anaerobic conditions to biogas.

2. It will help to reduce the use of fossil fuels and thus reduce CO2 emission.

Which of the statement(s) given above is/are correct?

a. 1 only

b. 2 only

c. Both 1 and 2

d. Neither 1 nor 2

3) Consider the following statements with respect to Eastern Economic Forum, 2021

1. It was established by the Russian Federation in 2015.

2. It is an international platform aiming to foster communication and cooperation between members from Russia and the Asia-Pacific region.

3. The main theme of this year‘s Forum is ‗New Opportunities for the Far East in a Changed World‘.

Which of the statement(s) given above is/are correct?

a. 3 only

b. 1 and 2 only

c. 2 and 3 only

d. 1, 2 and 3

4) Consider the following statements with respect to Phytoremediation

1. It is an autotrophic system powered by solar energy.

2. It is the process of using green plants to treat and control wastes in water, soil, and air.

Which of the statement(s) given above is/are correct?

a. 1 only

b. 2 only

c. Both 1 and 2

d. Neither 1 nor 2

5) Baira Siul Power Station is located in which of the following states?

a. Uttarakhand

b. Andhra Pradesh

c. Himachal Pradesh

d. Arunachal Pradesh

Answers

1. d

Climate Adaptive Planning for Resilience and Sustainability Development in Multi-Hazard Environment (CAP-RES) Programme

The Department of Science & Technology, Government of India (GoI), under the National Knowledge Mission on Climate Change (NKMCC), has supported the CAP-RES (Climate Adaptive Planning for Resilience and Sustainability) programme.

The mission is to be implemented by CECR-ECDRM division of National Institute of Disaster Management (NIDM).

NIDM works under the Ministry of Home Affairs.

CAP-RES aims to address the knowledge gap across the specified themes of resilience and sustainability.

This would enable effective interface of the science policy planning and practices and would facilitate knowledge based roadmap for addressing climate change induced risks and vulnerability.

Thematic Areas of CAP-RES include:

1. Green Growth and Disaster Risk Reduction,

2. Resilient Agriculture,

3. Resilient Health,

4. Climate Proofing Disaster Relief & Recovery, and

5. Environmental Policy Instruments in DRR

2. c

Biomethanation

Page 16: INDEX [iasparliament.s3.ap-south-1.amazonaws.com]

17

www.shankariasacademy.com | www.iasparliament.com

is a process by which organic material is microbiologically converted under anaerobic conditions to biogas.

Biomethanation has strong potential for the production of energy from organic residues and wastes.

It will help to reduce the use of fossil fuels and thus reduce CO2 emission.

3. d

Eastern Economic Forum 2021

The Eastern Economic Forum was established by decree of the President of the Russian Federation Vladimir Putin in 2015 to support the economic development of Russia‘s Far East and to expand international cooperation in the Asia-Pacific region.

The forum will meet annually in every September since 2015.

It will take place on 2–4 September in Vladivostok on the Far Eastern Federal University (FEFU) campus.

The Eastern Economic Forum was established by decree of the President of the Russian Federation Vladimir Putin in 2015 to support the economic development of Russia‘s Far East and to expand international cooperation in the Asia-Pacific region.

4. c

Phytoremediation

It is a plant-based approach, which involves the use of plants to extract and remove elemental pollutants or lower their bioavailability in soil.

Plants have the abilities to absorb ionic compounds in the soil even at low concentrations through their root system.

There are advantages of using phytoremediation, which include:

1. It is an autotrophic system powered by solar energy, therefore, simple to manage, and the cost of installation and maintenance is low

2. It can reduce exposure of the pollutants to the environment and ecosystem

3. It can be applied over a large-scale field and can easily be disposed

4. It prevents erosion and metal leaching through stabilizing heavy metals, reducing the risk of spreading of contaminants

5. It can also improve soil fertility by releasing various organic matters to the soil.

5. c

Recently, the National Hydroelectric Power Corporation (NHPC) Ltd, has indigenously renovated & modernized its 180 MW (MegaWatt) Baira Siul Power Station located in Chamba in Himachal Pradesh.

NHPC‘s first power station, the Baira Siul Power Station was under commercial operation from 1st April 1982 and had completed its useful life of 35 years.

Now, the renovation and modernization of all three units have been successfully completed and the life of Baira Siul Power Station has now been extended by another 25 years.

06-09-2021

1) Which of the following places is/are closely associated with Mahatma Gandhiji?

1. Mhow

2. Pietermaritzburg

3. Aga Khan Palace

4. Chaitya Bhoomi

Select the correct answer using the codes given below:

a. 3 only

b. 2 and 3 only

c. 3 and 4 only

d. 1, 2, 3 and 4

2) Which of the following statements regarding the Karbi-Anglong Peace Agreement is not correct?

a. It will transfer as much autonomy as possible in exercising their rights to the Karbi Anglong Autonomous Council.

b. A special development package of Rs. 1000 crore will be allocated by the Assam Government for the development of Karbi areas over five years.

Page 17: INDEX [iasparliament.s3.ap-south-1.amazonaws.com]

18

www.shankariasacademy.com | www.iasparliament.com

c. This agreement will ensure the protection of the culture, identity, language of the Karbi people and all-round development of the region.

d. The Consolidated Fund of the State of Assam will be amended to meet the resources of the Karbi Anglong Autonomous Council.

3) Which of the following Tiger Reserves in India are accredited with Conservation Assured | Tiger Standards (CA|TS) status?

1. Corbett Tiger Reserve

2. Anamalai Tiger Reserve

3. Mudumalai Tiger Reserve

4. Bandhavgarh Tiger Reserve

5. Ranthambore Tiger Reserve

Select the correct answer using the codes given below:

a. 1 and 4 only

b. 2 and 3 only

c. 1, 4 and 5 only

d. 1, 2, 3, 4 and 5

4) Which one of the following correctly describes the term, Breakthrough Infection, sometimes seen in the news recently?

a. It occurs when a ―variant of interest‖ becomes a ―variant of concern‖ in a very short period of time.

b. It is a condition in which a person gets infected with SARS-CoV-2 virus after 14 days of the second shot of vaccine.

c. It is an important discovery and recognition of a rapidly spreading unidentified variant of SARS-CoV-2 by WHO.

d. It is a condition where preventive steps taken to control a new variant have reduced the effectiveness related to previously circulating variants.

5) Consider the following statements with respect to Solar Storms

1. It is a directional ejection of a large mass of highly magnetised particles from the sun

2. Undersea cables have a higher risk of failure from solar storms compared to land cables.

Which of the statement(s) given below is/are correct?

a. 1 only

b. 2 only

c. Both 1 and 2

d. Neither 1 nor 2

6) Consider the following statements about Nipah Virus

1. It has an incubation period of 5-14 days.

2. It was first identified in Malaysia and first reported in West Bengal in India.

3. It can be transmitted through contaminated food.

Which of the statements given above is/are correct?

a. 1 only

b. 2 only

c. 1 and 2 only

d. 1, 2 and 3

7) Consider the following statements about World Conservation Congress

1. It is organised by the World Wide Fund for Nature once in 4 years.

2. The 2021 edition is organised by the France Government in Marseille, the second-largest city on its Mediterranean coast.

Which of the statements given above is/are correct?

a. 1 only

b. 2 only

c. Both 1 and 2

d. Neither 1 nor 2

8) Irrawaddy Dolphin can be found in which of the following regions?

1. Mekong River

2. Chambal River

3. Mahakam River

Page 18: INDEX [iasparliament.s3.ap-south-1.amazonaws.com]

19

www.shankariasacademy.com | www.iasparliament.com

4. Chilika Lake

5. Loktak Lake

Choose the correct option

a. 1 and 3 only

b. 1, 2 and 3 only

c. 1, 3 and 4 only

d. 1,2,3,4 and 5

9) Consider the following statements about Residents of UT of Ladakh and J&K

1. The resident certificate issued for the purpose of appointment to all the non-gazetted posts in the UT.

2. In Ladakh, it is restricted to only the Permanent Resident Certificate holders of the region whereas in J&K it is open to outsiders also.

Which of the statements given above is/are correct?

a. 1 only

b. 2 only

c. Both 1 and 2

d. Neither 1 nor 2

10) Consider the following statements about Red-Eared Slider Turtle

1. It is a native of South Asian Region.

2. It is one of the top 100 most invasive species in the world.

3. It is known to be a carrier of human-infected bacteria such as salmonella.

Which of the statements given above are correct?

a. 1 and 2 only

b. 1 and 3 only

c. 2 and 3 only

d. 1, 2 and 3

Answers

1. b

Mahatma Gandhiji

Porbandar, Gujarat – Birth Place of Mahatma Gandhi

Pietermaritzburg station, South Africa – It is the place where Gandhi was thrown out from the train compartment.

The incident changed his life, and he stood up against racial discrimination and made active non-violence as his mission.

Yerawda Jail, Pune – It is the place where historic Poona Act was signed when Gandhiji fasted to protest against Communal award.

He was housed three times in this jail, including in 1932 and in 1942 along with many other freedom fighters, during Quit India Movement.

Aga Khan Palace, Pune – This is the place where Gandhiji lived for 21 months after he gave the call for Quit India on 8th August, 1942.

During this period, the Mahatma lost his wife Kasturba and his secretary Narayan Desai.

The Samadhis of both of them are located in this grand palace of Pune.

Rajghat, Delhi – It is where the Samadhi of Mahatma Gandhi is located.

Mhow - Ambedkar was born in 1891 in the town and military cantonment of Mhow (now officially known as Dr Ambedkar Nagar) in the Central Provinces (now in Madhya Pradesh).

Chaitya Bhoomi – It is a Buddhist chaitya and the cremation place of B. R. Ambedkar, the chief architect of the Indian Constitution.

2. b

In the presence of Union Home Minister, Shri Amit Shah, a historic Karbi Anglong Agreement to end the decades old crisis ensuring Assam‘s territorial integrity was signed in New Delhi recently.

Salient Features of Karbi-Anglong Peace Agreement

Peace Accord – Under this historic agreement, 5 militant organizations laid down arms and more than 1000 of their armed cadres have given up violence and joined the mainstream of society in February 2021.

Page 19: INDEX [iasparliament.s3.ap-south-1.amazonaws.com]

20

www.shankariasacademy.com | www.iasparliament.com

Development Package – A special development package of Rs. 1000 crore will be allocated over five years by the Central Government and the Assam Government to take up special projects for the development of Karbi areas.

Transfer of Autonomy – This agreement will transfer as much autonomy as possible in exercising their rights to the Karbi Anglong Autonomous Council, without affecting the territorial and administrative integrity of Assam.

Protection of culture – This agreement will ensure the protection of the culture, identity, language, etc. of the Karbi people and all-round development of the region.

Rehabilitation of militants – A provision has also been made in this agreement to rehabilitate the cadres of armed groups.

Karbi Welfare Council – The Government of Assam will set up a Karbi Welfare Council to focus on the development of the Karbi people living outside the Karbi Anglong Autonomous Council area.

Funding support - The Consolidated Fund of the State will be amended to meet the resources of the Karbi Anglong Autonomous Council.

Overall, the present agreement proposes to give more legislative, executive, administrative and financial powers to the Karbi Anglong Autonomous Council.

3. b

CA|TS

CA|TS is globally accepted conservation tool, launched in 2013, that sets best practice and standards to manage tigers and encourages assessments to benchmark progress.

India‘s National Tiger Conservation Authority (NTCA) of the Ministry of Environment, Forest and Climate Change announced in 2020 the adoption of CA|TS across all the country‘s 50 tiger reserves.

The 50 tiger reserves are spread across 18 states covering an areas of over 72,000 km2 and contain over 70 per cent of the world‘s tiger population.

Only 14 tiger reserves in India has been accredited with the CA|TS tag.

The 14 tiger reserves are:

1. Manas, Kaziranga and Orang in Assam

2. Satpura, Kanha and Panna in Madhya Pradesh

3. Pench in Maharashtra

4. Valmiki Tiger Reserve in Bihar

5. Dudhwa in Uttar Pradesh

6. Sunderbans in West Bengal

7. Parambikulam in Kerala

8. Bandipur Tiger Reserve in Karnataka

9. Mudumalai and Anamalai Tiger Reserves in Tamil Nadu.

Well-known reserves like Corbett, Ranthambore and Bandhavgarh did not get the tag.

4. b

Breakthrough Infection

If a person gets infected with the SARS-CoV-2 virus, 14 days after the second shot of the vaccine, it is called a ‗breakthrough infection‘.

The two-week window is the time it takes for the body to produce necessary antibodies following a shot of the vaccine.

Breakthrough infections are not a surprise.

In clinical trials, all vaccines available have reported efficacy rates between 70% and 90%.

This implies that between 10% and 30% of a vaccinated population will be vulnerable to infection.

Variant of High Consequence – It has clear evidence that prevention measures or medical countermeasures (MCMs) have significantly reduced effectiveness relative to previously circulating variants.

5. c

Solar Storm

It is a directional ejection of a large mass of highly magnetised particles from the sun.

When the earth is in the direct path of such solar storms, these magnetised and charged

Page 20: INDEX [iasparliament.s3.ap-south-1.amazonaws.com]

21

www.shankariasacademy.com | www.iasparliament.com

solar particles will interact with the earth‘s magnetic field and induce strong electric currents on the earth‘s surface.

Hence, power grids, oil and gas pipelines, and networking cables are the most vulnerable.

Powerful solar superstorms can destroy long-distance undersea cables leading to massive Internet disruption lasting for several months.

Undersea cables have a higher risk of failure compared to land cables due to their large lengths as current is proportional to the area of the loop formed by the two grounds and the cable.

Communication satellites could also be among the severely affected systems due to direct exposure to highly charged particles in the storms.

The scale of impact is different for different regions.

Chinese cities are more prone to lose connectivity than Indian cities because the former connects to longer cables.

The U.S. is one of the most vulnerable locations with a high risk of disconnection from Europe during extreme solar events.

Europe is at a lower risk due to the presence of shorter land and undersea cables interconnecting the continent.

6. d

Nipah in Kerala

A new case of Nipah has been detected in the Kozhikode district.

The natural host of the Nipah virus are fruit bats of the Pteropodidae family and Pteropous genus, widely found in South and South East Asia.

However, the actual source of the current infection is not yet known.

It can be transmitted to humans from animals (such as bats or pigs), or contaminated foods and can also be transmitted directly from human-to-human.

There is no treatment or vaccine available for either people or animals. The primary treatment for humans is supportive care.

It was first recognized in 1999 during an outbreak among pig farmers in, Malaysia.

Till date, India has experienced four episodes of NiV outbreaks with CFR ranging from 65% to 100%. The evidence of NiV infection are reported in,

1. Siliguri district, West Bengal (2001),

2. Nadia district in West Bengal (2007),

3. Kozhikode district of Kerala (2018), followed by another outbreak in the same state in 2019.

7. b

World Conservation Congress

It is organised by International Union for Conservation of Nature (IUCN).

It is held once in 4 years.

It brings together several thousand leaders and decision-makers from government, civil society, indigenous peoples, business, and academia, with the goal of conserving the environment and harnessing the solutions nature offers to global challenges.

The 2021 edition is going to be organised by IUCN and the French government in Marseille.

It was originally scheduled for June 2020, was postponed due to the COVID-19 pandemic.

8. c

Irrawaddy Dolphin

Irrawaddy dolphins are found in coastal areas in South and Southeast Asia.

It is found in 3 rivers - The Ayeyarwady (Myanmar), the Mahakam (Indonesian Borneo) and the Mekong.

In India, it is found in Chilika Lake and coastal areas.

9. c

The Resident Certificate

It is for the purpose of appointment to all the non-gazetted posts.

The Ladakh administration has decided to issue ―Resident Certificate‖ only to the Permanent Resident Certificate holders of the region,

Page 21: INDEX [iasparliament.s3.ap-south-1.amazonaws.com]

22

www.shankariasacademy.com | www.iasparliament.com

According to the Jammu and Kashmir Grant of Domicile Certificate (Procedure) Rules, 2020, different categories of non-locals, including non-local government employees, to register for domicile certificates in J&K are eligible to apply for it.

10. c

Red-Eared Slider Turtle

It is a native of the southern U.S. (Mississippi river) and northern Mexico.

They live in still and warm waterbodies such as ponds, lakes, streams, and slow- running rivers.

It is identified as exotic and enlisted in the 100 most invasive species in the world.

They are considered a major threat to native turtle specials, as they mature fast, grow larger, and produce more offspring, and are very aggressive.

It has recently been found accidently from the Malankara dam in Idukki.

07-09-2021

1) Consider the following statements with respect to Green status of Species, consider the following statements:

1. It was developed by IUCN and adopted at the 25th UN Climate Change Conference (COP-25) held in 2019.

2. It assesses species and provides it with a Green Score ranging from 0-10, which shows how far a species is from its fully recovered state.

Which of the statement(s) given above is/are correct?

a. 1 only

b. 2 only

c. Both 1 and 2

d. Neither 1 nor 2

2) NAMASYA App was launched recently to empower which of the following?

a. AYUSH

b. Railways

c. Panchayati Raj

d. Micro and Small enterprise

3) With respect to Pollen, consider the following statements:

1. Pollen are considered major outdoor airborne allergens responsible for allergic rhinitis and asthma in humans.

2. Pollen distribution greatly depends on meteorological conditions, micro and macro topography of a particular area.

3. Scientists have developed a pollen calendar for Chandigarh, a first of its kind for an Indian city.

Which of the statement(s) given above is/are correct?

a. 1 and 2 only

b. 1 and 3 only

c. 2 and 3 only

d. 1, 2 and 3

4) Which one of the following best describes the term Animal Spirits in Economics?

a. It refers to the ability of humans to exchange one thing for another

b. It refers to an act of sustainable use of scarce economic resources for a long time

c. It describes how people tend to value an object more if they make or assemble it themselves

d. It refers to the emotions and instincts that guide the behaviour of investors in a market economy

5) Consider the following statements with respect to Inspiration4, sometimes seen in the news recently

1. It is the World‘s first all-civilian, non-governmental spaceflight to orbit around the Earth.

2. The Inspiration4 will orbit the Earth at 575km higher than the International Space Station and the Hubble space telescope.

Which of the statement(s) given above is/are correct?

a. 1 only

b. 2 only

Page 22: INDEX [iasparliament.s3.ap-south-1.amazonaws.com]

23

www.shankariasacademy.com | www.iasparliament.com

c. Both 1 and 2

d. Neither 1 nor 2

6) Consider the following statements about The Tribunals Reforms Bill, 2021

1. A Chairperson/member of a tribunal should have a minimum age of 50 years.

2. It provides for a 4-year term of office subject to the upper age limit of 70 years for the Chairperson and members.

Which of the statements given above is/are correct?

a. 1 only

b. 2 only

c. Both 1 and 2

d. Neither 1 nor 2

7) Consider the following statements about Manda Buffalo

1. It is a native breed of ancient Mandu region of Madhya Pradesh.

2. It is less prone to parasitic infection and both male and female buffaloes are used for ploughing.

Which of the statements given above is/are correct?

a. 1 only

b. 2 only

c. Both 1 and 2

d. Neither 1 nor 2

8) Which among the following is directly responsible for the monitoring of Asset Monetisation programme?

a. Department of Economic Affairs, Ministry of Finance

b. Department of Investment and Public Asset Management, Ministry of Finance

c. Department for Promotion of Industry and Internal Trade, Ministry of Commerce and Industry

d. None of the above

9) Consider the following statements about Deep Sea Mining

1. It is the process of retrieving mineral deposits from the area of the ocean below 2000 m.

2. International Tribunal for the Law of the Sea regulates all mineral-related activities in the international seabed area beyond the limits of national jurisdiction.

Which of the statements given above is/are correct?

a. 1 only

b. 2 only

c. Both 1 and 2

d. Neither 1 nor 2

10) Consider the following statements about Seaweed

1. They are marine non-flowering red algae found in the inter-tidal regions.

2. It reduces the requirement of fertilizers in agriculture.

3. Government has initiated 3000 tonnes of seaweed farming initiative in the island of Andaman and Nicobar and Gulf of Mannar region.

Which of the statements given above are correct?

a. 1 and 2 only

b. 2 and 3 only

c. 1 and 3 only

d. 1, 2 and 3

Answers

1. d

Green Status of Species

It was developed in response to Resolution- 041 of World Conservation Congress (WCC) held in 2012.

In this Resolution, IUCN called for the development of ‗Green Lists‘ of Species, Ecosystems and Protected Areas, in order to measure conservation success in these three areas.

Page 23: INDEX [iasparliament.s3.ap-south-1.amazonaws.com]

24

www.shankariasacademy.com | www.iasparliament.com

Since 2012, a Green List of Protected and Conserved Areas has been launched, and the development of the Green Status of Species began under the name ‗Green List of Species‘.

The Green Status assesses species against 3 essential facets of recovery:

1. Range - A species is fully recovered if it is present in all parts of its range, even those that are no longer occupied but were occupied prior to major human impacts/disruption;

2. Viability - It is viable (i.e., not threatened with extinction) in all parts of the range;

3. Functionality - It is performing its ecological functions in all parts of the range.

These factors contribute towards a ―Green Score‖ ranging from 0%-100% which shows how far a species is from its "fully recovered" state.

The IUCN Green Status classifies species into nine Species Recovery Categories, indicating the extent to which species are depleted or recovered compared to their historical population levels.

Each Green Status assessment measures the impact of past conservation on a species, a species‘ dependence on continuing support, how much a species stands to gain from conservation action within the next ten years, and the potential for it to recover over the next century.

The IUCN Red List describes how close a species is to extinction, but is not intended to paint a full picture of its status and functioning within its ecosystem.

With the IUCN Green Status, it becomes easy to track species recovery and dramatically improve our understanding of the state of the world‘s wildlife.

The IUCN Green Status of Species will be integrated into the IUCN Red List of Threatened Species, which will then provide a fuller picture of species‘ conservation status including both their extinction risk and recovery progress.

2. d

NAMASYA App

National Aluminium Company Ltd (NALCO) has recently launched the NALCO Micro and Small enterprise Yogayog (NAMASYA) Application.

NAMASYA is a modern & innovative bi-lingual platform developed exclusively for the benefit of the Company‘s Micro and Small Enterprise (MSE) Vendors.

The app empowers MSEs with required information about vendor registration process, items which can be supplied by them with technical specification, vendor development and training programmes of NALCO.

National Aluminium Company Ltd (NALCO)

It is a Navratna CPSE under the Ministry of Mines.

As a responsible Corporate and India‘s leading producer and exporter of alumina and aluminium, the Company has taken several initiatives towards easing the process of doing business, especially for the MSE sector involved in mining and metal business, and furthering inclusive growth and sustainable development in its ecosystem.

3. d

In a first-of-its kind in India, a team of researchers have conducted a study to examine the seasonal periodicities of airborne pollen spectrum and developed a pollen calendar for the city, Chandigarh.

The calendar identifies potential allergy triggers and provides a clear understanding for clinicians as well as allergy sufferers about their causes to help limit their exposure during high pollen loads.

Pollen

Pollens is released by plants; making millions of people suffer from pollen related allergic ailments.

They are considered major outdoor airborne allergens responsible for allergic rhinitis, asthma, and atopic dermatitis in humans.

They are known to be dispersed and transported to thousands of kilometers in the atmosphere.

Page 24: INDEX [iasparliament.s3.ap-south-1.amazonaws.com]

25

www.shankariasacademy.com | www.iasparliament.com

About 20-30% of the population suffers from allergic rhinitis/hay fever in India, and approximately 15% develop asthma.

Pollen allergy is recognized as one of the global health issues by the World Allergy Organization (WAO).

Pollen distribution greatly depends on meteorological conditions, micro and macro topography of a particular area.

Pollen Calendar

Pollen calendar represent the time dynamics of airborne pollen taxa in graphical form in a particular geographical area.

Pollen calendars are location-specific, with concentrations closely related to locally distributed flora.

Background

As per the Indian State Forest Report 2019 (ISRF), considering the percentage of the geographical area of State/Union Territory (UTs), the UT of Chandigarh has the highest percentage of tree cover, i.e., 22.34%.

However, an increase in green spaces will also lead to a rise in airborne pollen, consequently increasing pollen-related allergic ailments.

4. d

Animal Spirits

This is a term that refers to the emotions and instincts that guide the behaviour of investors and consumers in a market economy.

It was coined by British economist John Maynard Keynes in his 1936 book – ―The General Theory of Employment, Interest, and Money‖, to explain the persistence of economic fluctuations under capitalism.

Animal spirits refer to the ways that human emotion can drive financial decision-making in uncertain environments and volatile times.

Animal spirits essentially account for market psychology and in particular the role of emotion and herd mentality in investing.

Animal spirits are used to help explain why people behave irrationally, and are the forerunner to modern behavioral economics.

Keynes argued that investment and consumption are often based on how people feel about the overall economy rather than on unbiased, rational analysis of facts.

Critics have argued that while people are not perfectly rational, they are not completely guided by emotions either; hence, animal spirits cannot sufficiently explain economic cycles.

5. c

Inspiration4

SpaceX is going to launch ‗Inspiration4‘, the world‘s 1st all-civilian, non-governmental spaceflight soon.

This mission will take 4 private citizens into space in the Crew Dragon spacecraft, which will be privately operated by SpaceX.

This mission involves circling the Earth for 3 days at the height of 575km (low Earth orbit), and then splashing down into the Atlantic Ocean.

Inspiration4 will orbit the Earth at 575km, higher than the International Space Station (408km) and the Hubble space telescope (547km).

This will be the farthest distance travelled by a crewed mission since 2009, when astronauts went to repair the Hubble telescope.

The dome window has been inspired by the Cupola, a module on the International Space Station used to make observations about our planet.

Significance - The journey will present an opportunity for collecting many health data that will aid in planning future crewed space missions.

This will help in assessing behavioural and cognitive changes over the journey.

The travellers will undergo balance and prescription tests just before and after their journey to assess their response to the change in gravity.

6. a

The Tribunals Reforms Bill, 2021

Page 25: INDEX [iasparliament.s3.ap-south-1.amazonaws.com]

26

www.shankariasacademy.com | www.iasparliament.com

The Bill has been passed by the Lok Sabha and Rajya Sabha.

It seeks to dissolve certain existing appellate bodies and transfer their functions (such as adjudication of appeals) to other existing judicial bodies such as High Court and other civil courts.

The disputes under the following acts will be under High Court and their corresponding appellate bodies were dissolved

1. The Cinematograph Act, 1952

2. The Trade Marks Act, 1999

3. The Customs Act, 1962

4. The Patents Act, 1970

5. The Geographical Indications of Goods (Registration and Protection) Act, 1999

The Chairperson and members are appointed by the central government based on the recommendation of a search cum selection committee which consists of

i. The Chief Justice of India, or a Supreme Court Judge nominated by him, as the Chairperson (with casting vote),

ii. Two Secretaries nominated by the central government,

iii. The sitting or outgoing Chairperson, or a retired Supreme Court Judge, or a retired Chief Justice of a High Court, and

iv. The Secretary of the Ministry under which the Tribunal is constituted (with no voting right).

It provides for a four-year term of office subject to the upper age limit of 70 years for the Chairperson, and 67 years for members.

Further, it specifies a minimum age requirement of 50 years for appointment of a chairperson or a member.

7. b

Manda Buffalo

It is recognised as the 19th native breed of buffaloes in the country by ICAR-National Bureau of Animal Genetic Resources.

It is seen in the hill ranges of Eastern Ghats and plateau regions of Koraput region of Odisha.

They are less prone to parasitic infections.

It can live, produce and reproduce at low or no input system.

The average milk yield of these buffaloes is 2 to 2.5 litre at a time with more than 8 per cent fat.

It mature at around three years and drop the first calf at around 4 years.

Every 1.5 to 2 years they give birth to a calf for the whole life of around 20 years.

Its germplasm was first identified by the fisheries and animal resources development (FARD) department in collaboration with the Odisha university of Agriculture and Technology.

The recent recognition will lead to the efforts from Central and State government to conserve the breed and enhance its productivity through breeding strategy.

Other recognitions from Odisha –

i. 2 buffalo breeds (Chilika and Kalahandi)

ii. 4 cattle breeds (Binjharpuri, Motu, Ghumusari and Khariar)

iii. 1 sheep breed (Kendrapara)

8. d

Asset Monetisation programme

The Union government has announced its plans to ―monetize‖ about Rs 6 trillion worth of assets held by it, and public sector units (PSUs), under the National Monetisation Pipeline (NMP).

The Prime Minister‘s Office is directly monitoring the progress of the asset monetisation programme.

9. d

Deep Sea Mining

It is the process of retrieving mineral deposits from the deep sea – the area of the ocean below 200 m.

Parts of the ocean floor are rich in minerals, including polymetallic nodules composed mostly of copper, manganese, cobalt and nickel.

Page 26: INDEX [iasparliament.s3.ap-south-1.amazonaws.com]

27

www.shankariasacademy.com | www.iasparliament.com

These are found on seabeds 4 to 6 km below the surface.

International Sea Bed Authority is an intergovernmental body established by the Law of the Sea Convention to organize, regulate and control all mineral-related activities in the international seabed area beyond the limits of national jurisdiction.

10. a

Seaweed Production

Seaweeds are marine non-flowering plants with no stems, roots or leaves.

It is mainly found in the inter-tidal regions, in shallow and deep waters of the sea and also in estuaries and backwaters.

Large seaweeds form dense underwater forests known as kelp forests.

Currently, seaweed production in India is primarily confined to the Gulf of Mannar and Palk Bay in Tamil Nadu.

Government has initiated 3000-tonne seaweed farming in the island of Lakshadweep.

The indigenous red algae, Gracilaria edulis and Acanthophora spicifera are the species being farmed in the island.

08-09-2021

1) Consider the following statements with respect to PRANA portal

1. It is a portal for monitoring the implementation of National Clean Air Programme (NCAP).

2. It will support tracking of both physical as well as financial status of city air action plan implementation and disseminate information on air quality to public.

Which of the statement(s) given above is/are correct?

a. 1 only

b. 2 only

c. Both 1 and 2

d. Neither 1 nor 2

2) Which one of the following statements about National Clean Air Programme (NCAP) is not correct?

a. It proposes to reduce particulate matter (PM) pollution by 20-30% in at least 102 cities by 2024, with 2017 as the base year.

b. It was launched by the Ministry of Environment, Forests & Climate Change (MoEFCC).

c. Under the programme, non-attainment Cities are cities which frequently violated the Graded Response Action Plan (GRAP).

d. NCAP is not a legally binding document with any specified penal action against erring cities.

3) Gita Govinda is a 12th Century lyrical poem which describes the relationship between Krishna and Radha. The poem was composed by?

a. Harihara

b. Jayadeva

c. Bhavabhuti

d. Amarasimha

4) Consider the following statements with respect to 13th BRICS Summit

1. The event will be held under the Chairmanship of Russia.

2. The theme for the event is ―BRICS@15: Intra-BRICS cooperation for continuity, consolidation and consensus‖.

3. India hosted the BRICS Summit in 2012 and 2016.

Which of the statement(s) given above is/are correct?

a. 1 and 2 only

b. 1 and 3 only

c. 2 and 3 only

d. 1, 2 and 3

5) Blood Honey, sometimes seen in the news recently, refers to?

a. Honey collected by traditional honey gatherers of Sunderbans

Page 27: INDEX [iasparliament.s3.ap-south-1.amazonaws.com]

28

www.shankariasacademy.com | www.iasparliament.com

b. Artificial production of Indian Royal Jelly outside its natural habitat

c. A unique variety of honey in Thailand obtained from the blood of bees instead of its salivas

d. Highly profitable honey obtained through exploitation of traditional honey gathering tribes in Africa

6) Consider the following statements about Basmati Rice

1. It is grown in the foothills of the Himalayan region forming a part of the Indo-gangetic plain.

2. Punjab holds Geographical Indication tag for Basmati Rice.

3. On the global level, India and Pakistan are fighting for GI tag for it in the European Union.

Which of the statements given above is/are correct?

a. 2 only

b. 3 only

c. 1 and 3 only

d. 1 and 2 only

7) Consider the following statements about Dugong

1. It is an herbivorous mammal found in Gulf of Mannar, Gulf of Kutch, Andaman & Nicobar Islands and Lakshadweep in India.

2. India‘s first Dugong Conservation Reserve is going to come up in Palk Bay.

3. It is listed in Appendix I of CITES and Critically Endangered in IUCN red list of threatened species.

Which of the statements given above are not correct?

a. 1 and 2 only

b. 2 and 3 only

c. 1 and 3 only

d. 1, 2 and 3

8) Consider the following statements about Hycean Worlds

1. They are new class of exoplanets which are 2.6 times larger than Earth.

2. It has water rich interiors and hydrogen rich atmospheres.

Which of the statements given above is/are correct?

a. 1 only

b. 2 only

c. Both 1 and 2

d. Neither 1 nor 2

9) Consider the following statements about Dengue

1. It is a mosquito borne viral infection in which the vector species is same as chikungunya, yellow fever and Zika viruses.

2. There is an evidence of the possibility of maternal transmission.

3. It has become a leading cause for the death of children and adults in Asian and Latin American regions.

Which of the statements given above are correct?

a. 1 and 2 only

b. 2 and 3 only

c. 1 and 3 only

d. 1, 2 and 3

10) Consider the following statements about pattern of agricultural landholdings

1. More than 65% of the agricultural landholdings in the country belong to marginal farmers who owns less than 1 hectare of land.

2. According to the Agriculture Census 2015-16, the percentage of women landholders increased from 2010-11 t0 2015-16.

Which of the statements given above is/are correct?

a. 1 only

b. 2 only

c. Both 1 and 2

d. Neither 1 nor 2

Page 28: INDEX [iasparliament.s3.ap-south-1.amazonaws.com]

29

www.shankariasacademy.com | www.iasparliament.com

Answers

1. c

Portal for Regulation of Air-pollution in Non-Attainment cities (PRANA)

It is a portal for monitoring the implementation of National Clean Air Programme (NCAP).

It will support tracking of physical as well as financial status of city air action plan implementation and disseminate information on air quality management efforts under NCAP to public.

National Clean Air Programme (NCAP)

It has been launched by the Ministry of Environment, Forest and Climate Change as a comprehensive initiative in partnership with various Ministries and States to improve air quality at city, regional and national level.

It is a focused and time bound scheme to implement various sectoral policies, strengthen monitoring and enhance public participation in more than 100 cities for effective air quality management.

2. c

National Clean Air Programme

Objective - The overall objective of the programme includes comprehensive mitigation actions for prevention, control and abatement of air pollution.

It also aims to augment the air quality monitoring network across the country and strengthen the awareness and capacity building activities.

Also, city-specific action plans are being formulated for 102 non-attainment cities.

CPCB has identified list of polluted cities in which the prescribed National Ambient Air Quality Standards (NAAQS) are violated.

Target - It proposes a tentative national target of 20%-30% reduction in PM2.5 and PM10 concentrations by 2024, with 2017 as the base year for comparison.

However, the government has stressed that NCAP is a scheme, not a legally binding

document with any specified penal action against erring cities.

3. b

Gita Govinda

It is a work composed by the 12th-century Hindu poet, Jayadeva.

It describes the relationship between Krishna and Radhika.

The Gita Govinda consists of twelve chapters, further divided into twenty-four songs.

Each song consists of eight couplets, it is called Ashtapadi.

Jayadeva was a contemporary of several other poets in the court of Lakshmana Sena of Bengal.

4. c

13th BRICS Summit

As part of India‘s ongoing Chairship of BRICS in 2021, Prime Minister Shri Narendra Modi will chair the 13th BRICS Summit on 9 September 2021 in virtual format.

The theme for the event will be ‗BRICS @ 15: Intra-BRICS cooperation for continuity, consolidation and consensus‘.

This is the third time that India will be hosting the BRICS Summit after 2012 and 2016.

The 2020 BRICS summit was held virtually amid the covid pandemic under the chairmanship of Russia.

BRICS Background

The leaders of BRIC countries met for the first time at St Petersburg, Russia, on the margins of the G8 Outreach Summit in July 2006.

Shortly afterwards, in September 2006, the group was formalised as BRIC during the 1st BRIC Foreign Ministers' Meeting, which met on the sidelines of the General Debate of the UN Assembly in New York City.

After a series of high-level meetings, the 1st BRIC summit was held in Yekaterinburg, Russia on 16 June 2009.

5. a

Blood Honey

Page 29: INDEX [iasparliament.s3.ap-south-1.amazonaws.com]

30

www.shankariasacademy.com | www.iasparliament.com

Sunderbans being the home to the Royal Bengal tigers, every year people die venturing into the forests to collect this honey.

Due to the risks involved in its collection, it is called blood honey.

It is also called Sundarban honey, as it is collected by the Sunderbans‘ Moulis community.

This comparatively less thick honey has high demand for its nutritional value and purity.

As this honey is multi-floral and there is practically no use of antibiotics or pesticides in Sundarbans.

It is nearly twice as expensive as the branded honey.

Reasons for the high cost - One major part of the cost input is going deep into the forest area. This means that there are also no pesticides or chemicals or any other impurities in this honey.

Additionally, the purpose of ventures like this is the financial upkeep of this community too.

6. c

Basmati Rice

It is cultivated in the states of Himachal Pradesh, Punjab, Haryana, Delhi, Uttarakhand, Madhya Pradesh, Jammu and kashmir and western Uttar Pradesh.

In the domestic level, GI tag for Basmati Rice grown in Indo-gangetic plain is granted to Agricultural and Processed Foods Export Development Authority, a wing in the Ministry of Commerce and Industry.

The stakeholders from the Madhya Pradesh region have submitted documents for inclusion in the basmati grown areas.

On the global level, India and Pakistan are fighting for the GI tag for Basmati Rice in the European Union (EU).

International laws require that before applying for registration of any product in the international market it has to be protected under the geographical indication laws of that country.

Pakistan under its national law, has also granted GI tag for it.

7. c

Dugong Conservation Reserve

The Dugong, also called the sea cow, is an herbivorous mammal.

It grazes on sea grasses and comes to surface for breathe.

It can grow upto 3m long, 300kg weight and live for 65 to 70 years.

They are found in over 30 countries. In India, it is found in Gulf of Mannar, Gulf of Kutch, Palk Bay, and the Andaman and Nicobar Islands.

It is listed in Appendix I of CITES and Vulnerable in IUCN Status.

The Tamil Nadu State government recently announced that a 500-sqkm dugong conservation reserve will soon be set up in the Palk Bay.

8. c

Hycean Worlds

The word ―Hycean‖ represents the combination of Hydrogen and Ocean.

They are new class of exoplanets with densities between those of rocky super-Earths and larger mini-Neptunes

They are composed of water-rich interiors with massive oceans underlying hydrogen-rich atmospheres.

These planets can be up to 2.6 times larger than Earth and have atmospheric temperatures up to nearly 200 degrees Celsius.

Their oceanic conditions could be similar to those conducive for microbial life in Earth‘s oceans.

However, size alone is not enough to confirm whether a planet is Hycean. Other aspects such as mass, temperature and atmospheric properties are required for confirmation.

9. d

Dengue

Page 30: INDEX [iasparliament.s3.ap-south-1.amazonaws.com]

31

www.shankariasacademy.com | www.iasparliament.com

As many as 124 cases of dengue have been reported in Delhi between January 1 and September 4, the highest since 2018.

It is a mosquito-borne viral infection, found in tropical and sub-tropical climates worldwide, mostly in urban and semi-urban areas.

It is a severe, flu-like illness that affects infants, young children and adults, but seldom causes death.

It is transmitted by female mosquitoes mainly of the species Aedes aegypti and, to a lesser extent, Ae. albopictus.

These mosquitoes are also vectors of chikungunya, yellow fever and Zika viruses.

Dengue is caused by a virus of the Flaviviridae family and there are four distinct, but closely related, serotypes of the virus that cause dengue (DENV-1, DENV-2, DENV-3 and DENV-4).

Severe dengue affects most Asian and Latin American countries and has become a leading cause of hospitalization and death among children and adults in these regions.

There is evidence however, of the possibility of maternal transmission (from a pregnant mother to her baby).

10. c

Agricultural landholdings

68% of the agricultural landholdings in the country belong to the marginal (less than one hectare) category.

The share of the marginal category in total agricultureal landholding has been increasing over the years from 51% in 1970-71 to 68% in 2015-16.

The share of operational landholdings cultivated by women has increased from 11.7% in 2005-06 to 13.9% in 2015-16.

09-09-2021

1) Consider the following Statements about Mosquitoes

1. They are pollinators and its larvae are food for fishes, turtles and tadpoles.

2. They feed on mammals, reptiles, amphibians, fish and worms.

3. Arctic mosquitoes, a dominant source of food for birds are not human disease vectors.

Which of the statements given above are correct?

a. 1 and 2 only

b. 2 and 3 only

c. 1 and 3 only

d. 1, 2 and 3

2) Consider the following statements about FSSAI standards for Vegan Products

1. Vegan food is defined as foods that do not use any ingredients or additives of animal origin including honey related products.

2. Vegan food products should not be involved in animal testing for evaluating the safety of the final product or ingredient.

Which of the statement(s) given above is/are correct?

a. 1 only

b. 2 only

c. Both 1 and 2

d. Neither 1 nor 2

3) Consider the following statements with respect to Edible oil

1. India imports almost 90% of its edible oil requirements in which palm Oil alone accounts for 55% of imports.

2. Palm Oil is rich in Vitamin A and E and coenzymes like ubiquinone.

Which of the statement(s) given above is/are correct?

a. 1 only

b. 2 only

c. Both 1 and 2

d. Neither 1 nor 2

4) The government has come up with a production linked incentive (PLI) scheme for investors in Electrolyser manufacturing. In light of this, consider the following statements

Page 31: INDEX [iasparliament.s3.ap-south-1.amazonaws.com]

32

www.shankariasacademy.com | www.iasparliament.com

1. Electrolyzers are the machines that split water into hydrogen and oxygen.

2. They are used to produce green hydrogen using electricity produced from renewable energy sources.

3. At present, there are no electrolyzer manufacturers in India.

Which of the statement(s) given above is/are correct?

a. 1 and 2 only

b. 2 and 3 only

c. 1 and 3 only

d. 1, 2 and 3

5) Which of the following crops are considered for the rabi marketing season under the Minimum Support Price (MSP)?

1. Barley

2. Mustard

3. Soyabeen

4. Safflower

5. Sesamum

Choose the correct option

a. 1, 2 and 3 only

b. 1,2 and 4 only

c. 1, 3,4 and 5

d. 1,2,3, 4 and 5

Answers

1. d

Mosquitoes

Every year, mosquito-borne diseases kill more than a million people and infect almost one out of every 10 humans in the world.

They feed on mammals, reptiles, amphibians, fish and worms.

They are known pollinators but they visit flowers at night and there have not been enough studies into pollination at night.

Mosquito larvae are food for fishes, turtles and tadpoles and its adult population are preyed upon by lizards, frogs, salamanders, birds and bats.

It plays a big role in the arctic ecosystem. During summers, mosquitoes swarm the arctic in huge numbers. They are a dominant source of food for birds and they are not human disease vectors.

Parasites could be present in the gut of the mosquito or could move among human via the mosquito‘s saliva.

2. c

For the first time, FSSAI is looking at setting standards for the vegan food industry and use of a separate vegan food logo.

It framed the following guidelines regarding vegan food products.

Vegan food is defined as foods or ingredients that do not use any ingredients, additives and processing aids of animal origin including milk, fish, poultry, meat, egg or honey-related products.

It also should not contain materials of insect origin like silk, dyes, chitin/chitosan or ingredients that are clarified using animal sourced products.

It should not be involved in animal testing for evaluating the safety of the final product or ingredient.

It should also not contain any animal derived GMO (genetically modified organism) or use animal derived gene for manufacturing these products.

Traceability shall be established up to the manufacturer level.

Verification of the final product will need to be done through analytical tools to ascertain absence of animal origin material in the product.

3. c

Edible oil

India imports 60% of the edible oil it consumes and runs up an import bill of Rs.80,000 crore.

Page 32: INDEX [iasparliament.s3.ap-south-1.amazonaws.com]

33

www.shankariasacademy.com | www.iasparliament.com

Of that, palm oil alone accounts for 55%.

In the quest for self-sufficiency, the government introduced the National Mission for Edible Oil and Oil Palm (MNEO-OP), which targets to grow it on 3 million hectares over the next decade.

Palm oil is rich in vitamin A and E, and in coenzymes like ubiquinone.

It is the efficient source of vegetable oil.

4. d

Electrolysers

The government plans to bring in mandatory ‗green hydrogen purchase obligation‘ for the industry.

The industries are required to meet 10% of their requirements through domestic green hydrogen.

In order to meet the demand, it has come up with a production linked incentive (PLI) scheme for investors in Electrolyser manufacturing.

Electrolyzers are the machines that split water into hydrogen and oxygen.

They are used to produce green hydrogen using electricity produced from renewable energy sources such as solar and wind plants.

5. b

Minimum Support Price

Kharif Crops covered under the MSP – Paddy, Jowar, Bajra, Maize, Ragi, Arhar (Tur), Moong, Urad, Cotton, Groundnut in shell, Sunflower seed, Soyabean, Sesamum, Nigerseed

Rabi Crops covered under the MSP – Wheat, Barley, Gram, Masur (Lentil), Rapeseed & Mustard, Safflower, Toria

Other Crops – Copra, De-husked coconut, Raw Jute

Fair and Renumerative Price (FRP) for Sugarcane

11-09-2021

1) Consider the following statements with respect to INSPIRE Awards – MANAK Scheme

1. It aims to foster a culture of scientific innovation among school children in the age group of 10-15 years or studying in classes 6 to 10.

2. The scheme is being implemented jointly by the Department of Science and Technology and National Innovation Foundation.

3. The Scheme aligns with the action plan for Start-up India Initiative.

Which of the statement(s) given above is/are correct?

a. 1 only

b. 1 and 2 only

c. 1, 2 and 3

d. 2 and 3 only

2) Consider the following statements with respect to Hydrogen

1. Cost of Hydrogen produced from coal can be cheaper and less sensitive to imports compared to hydrogen produced through electrolysis and Natural Gas respectively.

2. Almost 100% of Hydrogen produced in India is through Natural Gas.

Which of the statement(s) given above is/are correct?

a. 1 only

b. 2 only

c. Both 1 and 2

d. Neither 1 nor 2

3) A recent carbon dating analysis of rice with soil, found in a burial urn in Tamil Nadu, has yielded the date of 1155 BC, thus dating back to 3,200 years.

The above description refers to which of the following Civilizations?

a. Porunai

b. Keezhadi

c. Kodumanal

Page 33: INDEX [iasparliament.s3.ap-south-1.amazonaws.com]

34

www.shankariasacademy.com | www.iasparliament.com

d. Gangaikonda Cholapuram

4) Consider the following statements with respect to Allen‘s Rule in Biology:

1. It states that within a broadly distributed taxonomic clade, populations and species of larger size are found in colder environments and smaller size are found in warmer regions.

2. Animals living in colder regions of the world have shorter limbs than those living in warmer regions as an adaptation to control the dissipation of heat.

Which of the statement(s) given above is/are correct?

a. 1 only

b. 2 only

c. Both 1 and 2

d. Neither 1 nor 2

5) Consider the following pairs:

Types of Hydrogen – Source

1. Blue Hydrogen – a. Water

2. Green Hydrogen – b. Natural Gas

3. Brown Hydrogen – c. Coal

Which of the pair(s) given above is/are correctly matched?

a. 1 and 3 only

b. 3 only

c. 1, 2 and 3

d. None of the above

Answers

1. c

INSPIRE Awards-MANAK (Million Minds Augmenting National Aspiration and Knowledge)

Aim - It aims to foster a culture of innovation and creative thinking amongst school students to address the societal need through Science and Technology.

Target Group - 10-15 years or Standard VI to X

Salient features – It help towards building a critical human resource pool for strengthening and expanding the science and technology system and increase the research and development base.

Target - Targeting one million original technological ideas/ innovations rooted in science and societal applications that solve daily life problems.

INSPIRE Award – MANAK aligns with the action plan for ―Start-up India‖ initiative launched by the Hon‘ble Prime Minister of India.

The shortlisted projects are also displayed at the National Level Exhibition and Project Competition (NLEPC).

Background

INSPIRE Scheme was launched in 2008 and has been implemented by the Department of Science and Technology (DST) since 2009-10.

Scheme covers students in the age group of 10-32 years in five different components.

In the context of the Start-up India initiative, INSPIRE Awards scheme has been revamped into INSPIRE Awards-MANAK.

2. c

Ministry of Coal has recently constituted 2 Committees to Prepare Road Map for Coal based Hydrogen Production.

Hydrogen

Coal is one of the important sources of hydrogen making (Brown Hydrogen).

However, Coal has not been encouraged elsewhere because of the fear that while extracting hydrogen via coal (from the moisture embedded in coal) there may be carbon emission.

Almost 100% of Hydrogen produced in India is through Natural Gas.

Cost of Hydrogen produced from coal can be cheaper and less sensitive to imports when compared with hydrogen production through electrolysis and Natural Gas respectively.

Page 34: INDEX [iasparliament.s3.ap-south-1.amazonaws.com]

35

www.shankariasacademy.com | www.iasparliament.com

3. a

A carbon dating analysis of rice with soil, found in a burial urn at Sivakalai in Thoothukudi district of Tamil Nadu, by the Miami-based Beta Analytic Testing Laboratory has yielded the date of 1155 BC, indicating that the Thamirabarani River (or) Porunai Civilisation dates back to 3,200 years.

Archaeological excavations are being done at Kodumanal, Keeladi, Korkai, Mayiladumparai, Sivakalai, Adichchanallur and Gangaikonda Cholapuram in Tamil Nadu.

Porunai River

The only major perennial river in Tamil Nadu, Thamirabarani has been mentioned as Porunai, Than Porunai, Porunal and Poruntham in Tamil literature — right from the Sangam era to Nayakar era (6th century BCE to 17th century CE).

4. b

Research says that warm-blooded animals maintain constant internal body temperature to prevent their bodies from overheating.

Elephants do it with their ear flaps, while birds regulate heat through their bills.

A recent study suggests that some have found another to cool down more quickly as a response to climate change: Shapeshifting.

A recent research by Deakins University examined the ways animal were changing the size of their ears, tails, beaks and other parts to deal with climate change and temperature rise.

Allen’s Rule

This refers to an eco-geographical rule which states that significant differences exist in the size of limbs and other external organs of animals, even within the same species, depending on the geographical region in which they live.

Animals living in colder regions of the world, for instance, have shorter limbs than those living in warmer regions as an adaptation to control the dissipation of heat.

A smaller body surface area helps animals in colder regions stay warm by slowing down the loss of body heat.

It is named after American biologist Joel Asaph Allen who proposed it in his 1877 paper ―The influence of physical conditions in the genesis of species‖.

Biological patterns such as Allen‘s Rule can help make predictions about how animals will evolve as the climate warms.

Bergmann’s Rule

It is an eco-geographical rule that states that within a broadly distributed taxonomic clade, populations and species of larger size are found in colder environments, while populations and species of smaller size are found in warmer regions.

In other words, as it gets colder, animals get bigger.

5. b

Page 35: INDEX [iasparliament.s3.ap-south-1.amazonaws.com]

36

www.shankariasacademy.com | www.iasparliament.com

13-09-2021

1) Consider the following statements regarding Account Aggregator (AA) Network, unveiled recently

1. It helps an individual securely access and share information from one financial institution to any other regulated financial institution.

2. An Account Aggregator is a type of RBI regulated entity.

3. Data cannot be shared without the consent of the individual.

Which of the statement(s) given above is/are correct?

a. 3 only

b. 1 and 3 only

c. 2 and 3 only

d. 1, 2 and 3

2) Which one of the following is the last-stand battle fought between the British Raj and Afghan tribesmen in 1897?

a. Battle of Chanderi

b. Battle of Purandar

c. Battle of Saragarhi

d. Battle of Haldighati

3) Swaraj Sastra is a book, which deals with the principles of a non-violent political order, written by?

a. Sarala Devi

b. Mahadev Desai

c. Jamnalal Bajaj

d. Acharya Vinoba Bhave

4) What is the scientific reason behind the usage of CO2 to extinguish the fire caused due to electrical or other fire accidents?

a. It reduces the heat generation around the accident spot

b. CO2 reacts with oxygen thereby reduces the source for the flame.

c. It is heavier than oxygen and covers fire like a blanket

d. The lighter nature of CO2 makes it to act as a catalyst to extinguish the fire

5) Which of the following crops can be grown in both kharif and Rabi seasons in India?

1. Rice

2. Wheat

3. Groundnut

4. Jowar

Select the correct answer using the code given below:

a. 1 and 2 only

b. 2 and 3 only

c. 1, 3 and 4 only

d. 1, 2, 3 and 4

Answers

1. d

India has recently unveiled the Account Aggregator (AA) network, a financial data-sharing system that could revolutionize investing and credit, giving millions of consumers greater access and control over their financial records and expanding the potential pool of customers for lenders and fintech companies.

Account Aggregator empowers the individual with control over their personal financial data, which otherwise remains in silos.

Account Aggregator (AA) Network is the first step towards bringing open banking in India and empowering millions of customers to digitally access and share their financial data across institutions in a secure and efficient manner.

Account Aggregator (AA)

An Account Aggregator is a type of RBI regulated entity (with an NBFC-AA license).

It helps an individual securely and digitally access and share information from one financial institution they have an account with to any other regulated financial institution in the AA network.

Page 36: INDEX [iasparliament.s3.ap-south-1.amazonaws.com]

37

www.shankariasacademy.com | www.iasparliament.com

Data cannot be shared without the consent of the individual.

There will be many Account Aggregators an individual can choose between.

Account Aggregator replaces the long terms and conditions form of ‗blank cheque‘ acceptance with a granular, step by step permission and control for each use of your data.

How AA is different from Aadhaar eKYC data sharing, credit bureau data sharing, and platforms like CKYC?

Aadhaar eKYC and CKYC only allow sharing of four ‗identity‘ data fields for KYC purposes (eg name, address, gender, etc).

Similarly, credit bureau data only shows loan history and/or a credit score.

The Account Aggregator network allows sharing of transaction data or bank statements from savings/deposit/current accounts.

2. c

Battle of Saragarhi

September 12, 2021 marks the 124th anniversary of the Battle of Saragarhi that has inspired a host of armies, books and films, both at home and abroad.

Saragarhi was the communication tower between Fort Lockhart and Fort Gulistan in the North West Frontier Province (NWFP).

Two forts, now in Pakistan, were built by Maharaja Ranjit Singh but renamed by the British.

Battle of Saragarhi was a last-stand battle fought between the British Raj and Afghan tribesmen (Afridi and Orakzai) in 1897.

During the Battle, 21 soldiers of 36th Sikhs (now 4 Sikh), led by Havildar Ishar Singh, along with a non-combatant called Daad were pitted against over 8,000 tribals but they managed to hold the fort for 7 hours.

Queen Victoria awarded to these 21 soldiers the Indian Order of Merit posthumously along with 2 ‗marabas‘ (50 acres) and Rs 500 each.

3. d

Acharya Vinoba Bhave wrote the Swaraj Shastra or A Grammar of Politics, which is ranked among the very few original treatises on politics.

4. c

5. c

Wheat needs 12 to 15 inches (31 to 38 centimeters) of water to produce a good crop.

It grows best when temperatures are warm, from 70° to 75° F (21° to 24° C), but not too hot and hence Wheat is grown as a Rabi crop and grown in the Kharif season.

Wheat also needs a lot of sunshine, especially when the grains are filling. Areas with low humidity are better since many wheat diseases thrive in damp weather.

14-09-2021

1) Groundswell Report is a series of reports published by?

a. World Bank Group

b. Food and Agricultural Organization

c. Sustainable Development Solutions Network

d. Office of the UN High Commissioner for Human Rights

2) Consider the following statements with respect to Software as a Service (SaaS)

1. It is a way of delivering applications in which software itself is considered as a service industry.

2. It allows each user to access software via Internet, instead of having to install the software on the user's computer.

3. It converted a company‘s fixed costs into variable costs and frees up capital for more high priority requirements.

Which of the statement(s) given above is/are correct?

a. 1 and 3 only

b. 3 only`

c. 1, 2 and 3

Page 37: INDEX [iasparliament.s3.ap-south-1.amazonaws.com]

38

www.shankariasacademy.com | www.iasparliament.com

d. 1 and 2 only

3) Consider the following statements:

1. He established a Provisional Government of India in Kabul in the middle of World War I in 1915.

2. He gave his own residence to establish the first technical school of India, named Prem Mahavidyalaya in 1909.

3. In 1932, he was nominated for the Nobel Peace Prize.

Who among the following personality is being described above?

a. Subramania Bharati

b. Maulana abul kalam Azad

c. Mylai Chinna Thambi Rajah

d. Raja Mahendra Pratap Singh

4) Climate & Clean Energy Agenda 2030 Partnership, sometimes seen in the news recently, is a joint initiative by India and?

a. France

b. United Arab Emirates

c. United Kingdom

d. United States of America

5) Species evenness refers to?

a. The variation is genes within a particular species.

b. The variety of living organisms on earth.

c. The measure of number of species found in a community

d. The measure of proportion of species at a given site.

Answers

1. a

The World Bank has recently released the second part of Groundswell report: Acting on Internal Climate Migration.

The Groundswell Report Series: Bridging the Gap

First Groundswell Report, 2018

It helps to understand the scale, trajectory, and spatial patterns of future climate migration within countries, with a focus on three regions: Sub-Saharan Africa, South Asia, and Latin America.

Specifically, it examined how slow-onset climate change impacts on water availability and crop productivity, and sea-level rise augmented by storm surge, could affect future internal migration.

Second Groundswell Report, 2021 (Acting on Internal Climate Migration)

It builds on the work of 1st report, applying the same approach to three new regions: the Middle East and North Africa, East Asia and the Pacific, and Eastern Europe and Central Asia.

Qualitative analyses of climate-related mobility in countries of the Mashreq and in Small Island Developing States (SIDS) are also provided.

Highlights of the report

According to the report, climate change can force some 216 million people in six world regions to move within their own countries by 2050.

Hotspots of internal climate migration can emerge as early as 2030 and continue to spread and intensify by 2050.

It also found that immediate and concerted action to reduce global emissions and support green, inclusive and resilient development, could reduce the scale of climate migration by as much as 80 per cent.

2. c

Software as a Service (SaaS)

Background

Earlier, companies would have to install software in their clients‘ localised hardware for them to use their applications.

So, this required customers to pay for use of the software upfront, and also for the hardware on which the software would run.

Page 38: INDEX [iasparliament.s3.ap-south-1.amazonaws.com]

39

www.shankariasacademy.com | www.iasparliament.com

Companies also had to invest in manpower/IT teams at their end to ensure successful implementation of the software and its continuous maintenance.

Software as a Service (SaaS)

It is a way of delivering software applications over the internet when a customer requires them.

In this system, software itself is considered as a service industry.

SaaS is one of three main categories of cloud computing, alongside infrastructure as a service (IaaS) and platform as a service (PaaS).

SaaS applications are sometimes called Web-based software, on-demand software, or hosted software.

Advantages - The biggest advantage is, it enables innovation.

It has basically converted fixed costs of companies into variable costs.

When capital is scarce, the SaaS model frees up capital for more high priority requirements.

3. d

Raja Mahendra Pratap Singh

He was a freedom fighter, revolutionary, writer, social reformer, and internationalist.

Raja Mahendra Pratap Singh was born into the ruling Jat family of Mursan estate in Hathras in 1886.

In 1907, the young Raja proceeded on a world tour with his wife, who was Sikh.

On his return, the Raja gave up his own residence in Mathura to be converted into a technical school named Prem Mahavidyalaya in 1909.

It is said to have been the country‘s first polytechnic.

Like many others in the region, the family contributed to Sir Syed‘s efforts to set up the university.

The family is said to have given land to AMU, some parts of which were donation, while other parts were granted on lease.

On December 1, 1915, he proclaimed the first Provisional Government of India outside India at the historic Bagh-e-Babur in Kabul.

He declared himself president, and his fiery fellow revolutionary Maulana Barkatullah of Bhopal, prime minister, of the Provisional Government.

In 1929, Mahendra Pratap launched the World Federation in Berlin.

He was nominated for the 1932 Nobel Peace Prize by the Swedish doctor N A Nilsson, who was a member of the Commission of the Permanent International Peace Bureau.

4. d

Climate Action & Finance Mobilization Dialogue

India and the USA launched the ―Climate Action and Finance Mobilization Dialogue (CAFMD)‖.

CAFMD was launched under India-US Climate and Clean Energy Agenda 2030 Partnership launched at the Leaders' Summit on Climate.

It aims to advance inclusive and resilient economic development.

India-US Climate & Clean Energy Agenda 2030 Partnership

It is a joint climate and clean energy initiative by India with the US.

It will demonstrate how the world can align towards swift climate action with inclusive & resilient economic development by taking into account the national circumstances as well as sustainable development priorities.

The objectives of the initiative are,

1. To mobilise investments and speed clean energy deployment,

2. To demonstrate & scale clean technologies needed to decarbonize sectors including industry, transportation, power, and buildings,

3. Build capacity to measure, manage, and adapt to the risks of climate-related impacts,

4. Enabling green collaborations in India in order to create templates of

Page 39: INDEX [iasparliament.s3.ap-south-1.amazonaws.com]

40

www.shankariasacademy.com | www.iasparliament.com

sustainable development for other developing countries.

The Partnership will proceed along two main tracks:

1. Strategic Clean Energy Partnership and

2. Climate Action and Finance Mobilization Dialogue.

5. d

Species evenness is how evenly spread the population is across the species in an area- if you have 100 elephants and 105 lions, the population has high evenness but if you had 200 lions and 5 elephants, even though the number of animals is the same, the population is not very even. So here, there is species unevenness.

Statement 1: The variation is genes within a particular species – genetic variation

Statement 2: The variety of living organisms on earth – biodiversity

Statement 3: Species diversity is a measure of community complexity. It is a function of both the number of different species in the community (species richness) and their relative abundances (species evenness).

15-09-2021

1) Oxybenzone, Octinoxate and Octocrylene are knowns as Toxic 3 Os. Which of the following statements are correct with respect to Toxic 3 Os?

1. Toxic 3 Os are active ingredients present in sunscreens, which can trigger health risks to people and marine life.

2. Octocrylene is particularly toxic to corals which may severely damage or kill coral.

Select the correct answer using the codes given below:

a. 1 only

b. 2 only

c. Both 1 and 2

d. Neither 1 nor 2

2) Consider the following statements with respect to T+1 Trade Settlement Period

1. It reduces the number of outstanding unsettled trades at any instant, and will reduce systemic risk.

2. If the stock exchange opts for this it will have to mandatorily continue with it for a minimum 6 months.

Which of the statement(s) given above is/are correct?

a. 1 only

b. 2 only

c. Both 1 and 2

d. Neither 1 nor 2

3) Which of the following statements is/are not correct with respect to Methanol?

1. It is a low carbon, hydrogen carrier fuel produced from high ash coal and natural gas.

2. It is used as a motor fuel to power ship engines, and to generate clean power all over the world.

Select the correct answer using the codes given below:

a. 1 only

b. 2 only

c. Both 1 and 2

d. Neither 1 nor 2

4) Which of the following statements is/are not correct with respect to Folk varieties of Rice?

1. Tilak Chandan is a thick, small-grained rice famous for its fragrance and cultivated in Uttar Pradesh‘s Bijnore and Rampur.

2. Bindli is a widely cultivated folk rice variety of India, which can be cultivated even under dry condition.

Select the correct answer using the codes given below:

a. 1 only

b. 2 only

c. Both 1 and 2

d. Neither 1 nor 2

Page 40: INDEX [iasparliament.s3.ap-south-1.amazonaws.com]

41

www.shankariasacademy.com | www.iasparliament.com

5) Which of the following statements are correct with respective to Terai Arc Landscape

1. It is a stretch between river Yamuna in the west and river Bhagmati in the east.

2. It is restricted only to India that is spread across the region of Madhya Pradesh, Haryana & Uttar Pradesh.

Select the correct answer using the codes given below:

a. 1 only

b. 2 only

c. Both 1 and 2

d. Neither 1 nor 2

Answers

1. a

Toxic 3 Os

Oxybenzone, Octinoxate and Octocrylene are known as Toxic 3 Os , which active ingredients present in more than two-thirds of all sunscreens.

They pose a threat to public health, marine life and coral reefs, according to latest research.

Octocrylene in sun protection products degrades into benzophenone, a carcinogen that can also interfere with key hormones and reproductive organs.

They cause human cell damage that has been linked to cancer, disrupt hormones, have been found in breast milk, blood and urine and are known allergens.

Oxybenzone is particularly toxic to corals at concentrations as low as a few parts per trillion, the equivalent of three drops in an Olympic-size swimming pool may be enough to severely damage or kill coral.

2. c

T+1 Settlement

In the securities industry, the trade settlement period refers to the time between the trade date that an order is executed in the market and the settlement date when a trade is considered final.

On the last day of the settlement period, the buyer becomes the holder of record of the security.

In T+1, settlement of the trade takes place in one working day and the investor will get the money on the following day.

In T+2, if an investor sells shares, the settlement of the trade takes place in two working days (T+2) and the broker who handles the trade will get the money on the third day, but will credit the amount in the investor‘s account only by the fourth day.

In effect, the investor will get the money only after three days.

Recently, Securities and Exchange Board of India (SEBI) allowed stock exchanges to start the T+1 system as an option in place of T+2 for completion of share transactions.

If the stock exchange opts for the T+1 settlement cycle for a scrip, it will have to mandatorily continue with it for a minimum 6 months.

A scrip is a substitute or alternative to legal tender that entitles the bearer to receive something in return.

3. d

Methanol

Methanol is a low carbon, hydrogen carrier fuel produced from high ash coal, agricultural residue, CO2 from thermal power plants and natural gas.

Although slightly lower in energy content than petrol and diesel, methanol can replace both these fuels in the transport sector (road, rail and marine), energy sector (comprising boilers, process heating modules, tractors and commercial vehicles) and retail cooking (replacing LPG [partially], kerosene and wood charcoal).

Methanol is utilized as a motor fuel, to power ship engines, and to generate clean power all over the world.

However, the majority of worldwide methanol production is derived from natural gas, which is a relatively easy process.

Page 41: INDEX [iasparliament.s3.ap-south-1.amazonaws.com]

42

www.shankariasacademy.com | www.iasparliament.com

Recently, the first Indigenously Designed High Ash Coal Gasification Based Methanol Production Plant has been opened in Hyderabad.

4. b

Folk Rice varieties

Tilak Chandan is a thick, small-grained rice famous for its fragrance and cultivated in Uttar Pradesh‘s Bijnore and Rampur.

Bindli, a once famous rice variety of Uttar Pradesh is now virtually out of cultivation, with only a few farmers cultivating it in the plains of Pauri district, Uttarakhand.

It can be cultivated under rain-fed, irrigated as well as waterlogged conditions.

Kalanamak, a scented rice variety grown in Uttar Pradesh, is fast going out of cultivation.

Samples of 94 landraces of rice or folk varieties were sourced from different geographical regions of India and conserved in the germplasm bank of Basudha Farm, Rayagada district, Odisha.

5. a

Terai Arc Landscape

It is an 810 km stretch between the river Yamuna in the west and the river Bhagmati in the east.

It is spread across the Indian states of Uttarakhand, Uttar Pradesh and Bihar, and the low lying hills of Nepal.

It boasts of some of India‘s most well-known Tiger Reserves and Protected Areas such as Corbett Tiger Reserve (Uttarakhand), Rajaji National Park (Uttarakhand), Dudhwa Tiger Reserve (Uttar Pradesh), and Valmiki Tiger Reserve (Bihar).

The forests are home to three flagship species, the Bengal tiger (Panthera tigris), the greater one horned rhino (Rhinoceros unicornis) and the Asian elephant (Elephas maximus).

16-09-2021

1) Shoonya Campaign, sometimes seen in the news recently, is associated with?

a. A Roadmap to zero new HIV infections

b. To promote zero pollution delivery vehicles

c. To achieve zero malnutrition related deaths in India

d. Awareness program about India‘s net zero emission targets

2) Consider the following statements with respect to Youth Co:Lab

1. It was co-created in 2017 by the United Nations Development Programme and Citi Foundation.

2. It aims to establish a common agenda for countries in the Asia-Pacific region to empower and invest in youth.

Which of the statement(s) given above is/are correct?

a. 1 only

b. 2 only

c. Both 1 and 2

d. Neither 1 nor 2

3) Consider the following statements with respect to Grindadrap, sometimes seen in the news recently

1. It is a traditional annual practice of sea mammal hunting practiced in Faroe Islands, Denmark.

2. The hunt usually focuses on whales rather than dolphins.

Which of the statement(s) given above is/are correct?

a. 1 only

b. 2 only

c. Both 1 and 2

d. Neither 1 nor 2

4) Reforms in Urban Planning Capacity in India Report was launched recently by?

a. NITI Aayog

b. UN- Habitat

c. Prime Minister‘s Office

d. Ministry of Housing & Urban Affairs

Page 42: INDEX [iasparliament.s3.ap-south-1.amazonaws.com]

43

www.shankariasacademy.com | www.iasparliament.com

5) In order for bio magnification to occur, the pollutant must be:

1. Long-lived

2. Static

3. Soluble in water

4. Biologically active

Select the correct answer using the code given below:

a. 1, 2 and 3 only

b. 1 and 4 only

c. 1, 2, 3 and 4

d. 3 and 4 only

Answers

1. b

With Rocky Mountain Institute (RMI) and RMI India‘s support, NITI Aayog has launched Shoonya Campaign.

Shoonya Campaign

The Shoonya campaign aims to:

1. Accelerate adoption of electric vehicles (EVs) in the urban deliveries segment and

2. Create consumer awareness about the health, environmental and economic benefits of zero-pollution delivery.

Urban freight vehicles account for 10% of freight transportation-related CO2 emissions in India, and these emissions may grow by 114% percent by 2030.

As part of the campaign, a corporate branding & certification programme is being launched to promote industry‘s efforts towards transitioning to EVs for final-mile deliveries.

An online tracking platform will share the campaign‘s impact through data such as vehicle kilometers electrified, carbon savings, criteria pollutant savings and other benefits from clean delivery vehicles.

2. c

Youth Co:Lab

It was co-created in 2017 by the United Nations Development Programme (UNDP) and the Citi Foundation.

It positions young people front and center in order to solve the region‘s most pressing challenges.

It aims to establish a common agenda for countries in the Asia-Pacific region to empower and invest in youth, so that they can accelerate the implementation of the Sustainable Development Goals (SDGs) through leadership, social innovation and entrepreneurship.

In India

In India, Youth Co:Lab has been launched by UNDP, in collaboration with Atal Innovation Mission, NITI Aayog in 2019.

Since then, three editions of Youth Co:Lab, including youth social innovation challenges, supporting enterprises, and youth focused dialogues have been jointly implemented.

Youth Co:Lab India, 2021 focuses on Circular Economy, Sustainable Transportation, Sustainable Tourism and Sustainable Food Tech.

This year the theme will focus on youth led innovation for climate action and accelerating green recovery.

3. c

Grindadrap

It is a traditional annual killing of whales and dolphins.

It has been going on for over four hundred years on the small North Atlantic island – Faroe Island that belong to Denmark.

The animals are driven into shallow waters where they are then cut with knives.

The killing is regulated by local laws.

The meat and blubber from the animals is shared out among the community.

Neither the pilot whales nor the white-sided dolphins are endangered.

The hunt usually focuses on the whales rather than the dolphins.

Page 43: INDEX [iasparliament.s3.ap-south-1.amazonaws.com]

44

www.shankariasacademy.com | www.iasparliament.com

4. a

NITI Aayog has recently launched a report on ‗Reforms in Urban Planning Capacity in India‘.

NITI Aayog had constituted an Advisory Committee on ‗Reforms in Urban Planning Education in India‘ in October 2020.

The Committee has concluded its mandate with this report.

The report comprises a set of recommendations on various aspects of urban planning such as interventions for planning healthy cities, optimum utilization of urban land, ramping up human-resource capacities, strengthening urban governance, building local leadership, enhancing the role of private sector, and advancing the urban planning education system.

5. b

The pollutant must be:

1. Long lived – so that it would stay for long duration

2. Mobile – so that it would move from one organism to other

3. Soluble in fat – so that It would not be decamped/digested

4. Biologically active

Example: DDT, PCB

Heavy Metals: Copper, Mercury, Cadmium, Lead, Zinc

17-09-2021

1) Which of the following correctly describes R Corona Borealis?

a. Hydrogen-deficient Star

b. High precision parallax collecting Satellite

c. Bright dancing lights in Northern Hemisphere

d. A small constellation in the Northern Celestial Hemisphere

2) Consider the following statements with respect to AUKUS Agreement

1. It is a trilateral security partnership between France, the United Kingdom and the United States.

2. Under the Agreement the United States will help Australia to deploy nuclear-powered submarines in the Pacific region.

3. The deal explicitly mentions that it is a step towards curtailing China, especially in and around the South China Sea.

Which of the statement(s) given above is/are correct?

a. 1 only

b. 2 only

c. 1 and 2 only

d. 1, 2 and 3

3) With respect to Low-C bricks, sometimes seen in the news recently, consider the following statements:

1. These are low carbon bricks produced using construction and demolition (C&D) wastes and alkali-activated binders.

2. They are manufactured either through high-temperature firing or using high-energy carbon binders such as Portland cement.

Which of the statement(s) given above is/are correct?

a. 1 only

b. 2 only

c. Both 1 and 2

d. Neither 1 nor 2

Page 44: INDEX [iasparliament.s3.ap-south-1.amazonaws.com]

45

www.shankariasacademy.com | www.iasparliament.com

4) Baijayant Panda Committee, sometimes seen in the news recently, was constituted?

a. To suggest measures to safeguard children from online gambling frauds

b. To create a skeletal framework on Digital Ecosystem for Agriculture in India

c. To review National Cadet Corps in order to make it more relevant in changed times

d. To examine the feasibility of extending Production Linked Incentive Scheme to various sectors

5) Which of the following entity has recently issued the first ever Euro-denominated Green bond from India?

a. Indian Railways

b. Reserve Bank of India

c. Power Finance Corporation

d. International Financial Services Centre

Answers

1. a

R Corona Borealis

R Corona Borealis is a low-mass yellow supergiant star in the constellation of Corona Borealis.

Corona Borealis is a small constellation in the Northern Celestial Hemisphere.

R Coronae Borealis stars (RCB), have very little hydrogen and are often referred as Hydrogen-deficient star.

It is about 90% helium and less than 1% hydrogen.

The majority of the remainder is carbon.

This is in stark contrast to the majority of stars whose atmospheres are dominated by hydrogen.

2. b

The United States, United Kingdom and Australia has recently announced a new defence deal, under which America and Britain will help Australia deploy nuclear-powered submarines in the Pacific region.

AUKUS Agreement

It is a new enhanced trilateral security partnership between United States, United Kingdom and Australia.

It aims to create a partnership where technology, scientists, industry, defence forces are all working together to deliver a safer and more secure region that ultimately benefits all.

The first major initiative of AUKUS would be to deliver a nuclear-powered submarine fleet for Australia.

Australia will be only the second country after Britain in 1958 to be given access to US nuclear technology to build nuclear-powered submarines.

The nuclear-powered submarines will give Australia naval heft in the Pacific, where China has been particularly aggressive.

Although the leaders of these three countries did not say so, the deal is seen as a step towards curtailing China, which has made significant aggressive manoeuvres in the Pacific region, especially in and around the South China Sea, where it has expansive territorial claims.

The AUCKUS & France

The deal has complicated the relations between France and Australia, and also France and the US.

Back in 2016, Australia had signed a contract to buy 12 Attack-class submarines from the French shipbuilder Naval Group.

With this new agreement, France-Australia submarine deal seems to be collapsed and a harsh legal battle over the contract appears inevitable.

France is upset as it has been kept out of the loop and also called the partnership, ―a stab in the back‖.

3. a

Background

Conventionally, building envelopes consist of masonry walls built with burnt clay bricks, concrete blocks, hollow clay blocks, fly ash bricks, lightweight blocks, and so on.

Page 45: INDEX [iasparliament.s3.ap-south-1.amazonaws.com]

46

www.shankariasacademy.com | www.iasparliament.com

The envelopes spend energy during their production, thus incurring carbon emission (i.e., possess embodied carbon) consume mined raw material resources which lead to unsustainable constructions.

The masonry units are manufactured either through the process of firing or using high-energy/embodied carbon binders such as Portland cement.

As a result, the annual consumption of bricks and blocks in India is about 900 million tonnes.

Besides, the construction industry generates vast amounts (70 – 100 million tonnes per annum) of construction and demolition waste (CDW).

Low-C bricks

The team of researchers from Indian Institute of Science (IISc), Bangalore developed low embodied carbon bricks from construction and demolition (C&D) waste through an alkali activation process using fly ash and ground slag and characterising the thermal, structural, and durability characteristics of Low-C bricks and their masonry.

It does not require high-temperature firing, and avoid the use of high-energy materials such as Portland cement.

The technology will also solve the disposal problems associated with C&D waste mitigation.

4. c

Ministry of Defence has recently constituted a High Level Expert Committee, under the chairmanship of former Member of Parliament Shri Baijayant Panda, for a comprehensive review of National Cadet Corps (NCC) in order to make it more relevant in changed times.

Indian Cricketer MS Dhoni, Mahindra Group Chairman Anand Mahindra is among the members of the committee.

National Cadet Corps (NCC)

The NCC is the largest uniformed organisation which aims at developing character, discipline, a secular outlook and ideals of selfless service amongst young citizens.

It also aims to create a pool of organised, trained and motivated youth with leadership qualities in all walks of life.

5. c

Euro-denominated Green Bond

Power Finance Corporation Ltd (PFC), the leading NBFC in power sector, has successfully issued its maiden Euro 300 million 7-year Euro Bond.

It is the first ever Euro denominated Green bond issuance from India.

Moreover, it is the first ever Euro issuance by an Indian NBFC and the first Euro bond issuance from India since 2017.

The issuance saw a strong participation from institutional investors across Asia and Europe with participation from across 82 accounts and was oversubscribed 2.65 times.

18-09-2021

1) The Five Eyes is an intelligence alliance comprising which of the following countries?

1. Canada

2. Australia

3. New Zealand

4. The United States

5. The United Kingdom

Select the correct answer using the codes given below:

a. All except 1

b. All except 3

c. All except 1 and 2

d. 1, 2, 3, 4 and 5

2) Consider the following statements:

1. Generally, Indian coal has high ash content.

2. In India, coking coal dominates the total coal resources than non-coking coal.

3. Out of the total lignite reserves in India, 90% occur in the southern State of Tamil Nadu.

Which of the statements given above are correct?

Page 46: INDEX [iasparliament.s3.ap-south-1.amazonaws.com]

47

www.shankariasacademy.com | www.iasparliament.com

a. 1 and 2 only

b. 1 and 3 only

c. 2 and 3 only

d. 1, 2 and 3

3) Consider the following statements:

1. The Guptas were the first to build structural temples.

2. Dashavatara Temple in Uttar Pradesh is one of the structural temples built during Guptas.

Which of the statement(s) given above is/are correct?

a. 1 only

b. 2 only

c. Both 1 and 2

d. Neither 1 nor 2

4) Consider the following statements with respect to Shankhalipi Script

1. It is a term used by scholars to describe ornate spiral characters assumed to be Brahmi derivatives that look like conch shells.

2. They are found in inscriptions across north-central India and dated between 4th and 8th centuries.

3. Both Shankhalipi and Brahmi are stylised scripts used primarily for names and signatures.

Which of the statement(s) given above is/are correct?

a. 1 and 2 only

b. 1 and 3 only

c. 2 and 3 only

d. 1, 2 and 3

5) The Barak River is one of the major rivers of the Surma-Meghna River System of Bangladesh. It passes through which of the following states in India before entering into Bangladesh?

1. Manipur

2. Mizoram

3. Assam

4. Tripura

Select the correct answer using the code given below

a. 1 and 2 only

b. 1, 2 and 3 only

c. 1, 2 and 4 only

d. 1, 2, 3 and 4

Answers

1. d

The United States of America seeks to invite South Korea to join the ‗Five Eyes‘ intelligence-sharing alliance.

A proposal to add four countries including South Korea, Japan, India and Germany is being discussed in the US Senate.

The Five Eyes

The Five Eyes is an intelligence-sharing alliance consisting of the US, UK, Australia, Canada & New Zealand.

The origins of the Five Eyes can be traced back to the informal meetings between the US and UK code-breakers during the Second World War.

It evolved during the Cold War as a mechanism for monitoring the Soviet Union and sharing classified intelligence.

It is often described as the world's most successful intelligence alliance.

2. b

India has recently developed an indigenous technology to convert high ash Indian coal to methanol and established its first pilot plant in Hyderabad.

This technology will help the country move towards the adoption of clean technology and promote the use of methanol as a transportation fuel (blending with petrol), thus reducing crude oil imports.

Indian Coal

Coal is the most important and abundant fossil fuel in India.

Page 47: INDEX [iasparliament.s3.ap-south-1.amazonaws.com]

48

www.shankariasacademy.com | www.iasparliament.com

It accounts for 55% of the country's energy need.

The country's industrial heritage was built upon indigenous coal.

In India coal is broadly classified into two types

1. Coking coal

2. Non-coking coal

The Coking coal constitutes only a small part of the total coal resources of the country.

As per Geological Survey of India, the reserve of coking coal (prime, medium and semi-coking) was 35,004 MT and non-coking coal was 3,09,017 MT.

Hard coal deposit spread over 27 major coalfields, are mainly confined to eastern and south central parts of the the country.

The lignite reserves stand at a level around 36 billion tonnes, of which 90 % occur in the southern State of Tamil Nadu.

3. c

Gupta Architecture

The Archaeological Survey of India (ASI) has recently discovered the remains of an ancient temple dating back to the Gupta period (5th century) in a village in Uttar Pradesh‘s Etah district.

The stairs of the temple had ‗shankhalipi‘ inscriptions, which were deciphered by the archaeologists as saying, ‗Sri Mahendraditya‘, the title of Kumaragupta I of the Gupta dynasty.

As per the ASI, the stairs led to a structural temple built during the Gupta period.

The discovery becomes significant since only two other structural temples from the Gupta age have been found so far — Dashavatara Temple (Deogarh) and Bhitargaon Temple (Kanpur Dehat).

In the 5th century, Kumaragupta I ruled for 40 years over north-central India.

The Guptas were the first to build structural temples, distinctly different from the ancient rock-cut temples.

4. d

Shankhalipi Script

Shankhalipi or ‗shell-script‘ is a term used to describe ornate spiral characters assumed to be Brahmi derivatives that look like conch shells or shankhas.

They are found on temple pillars, columns and rock surfaces across north-central India and they date to between the 4th and 8th centuries.

Both Shankhalipi and Brahmi are stylised scripts used primarily for names and signatures.

The inscriptions consist of a small number of characters, suggesting that the shell inscriptions are names or auspicious symbols or a combination of the two.

5. b

Barak River

The Barak River is one of the major rivers of South Assam and is a part of the Surma-Meghna River System.

It rises in the hill country of Manipur State, where it is the biggest and the most important of the hill country rivers.

After Manipur it flows through Mizoram State and into Assam, ending just after it enters Bangladesh where the Surma and Kushiyara rivers begin.

21-09-2021

1) Consider the following statements

1. Virtual Reality is an enhanced version of the real physical world that is achieved through the use of digital visual elements.

2. Augmented Reality requires special equipment such as computers, sensors, headsets, and gloves, while Virtual Reality requires only a smart phone.

Which of the above statement(s) is/are true?

a. 1 only

b. 2 only

c. Both 1 and 2

d. Neither 1 nor 2

Page 48: INDEX [iasparliament.s3.ap-south-1.amazonaws.com]

49

www.shankariasacademy.com | www.iasparliament.com

2) Consider the following statements about the Global Innovation Index 2021

1. India‘s rank in this index has been consistently rising since 2015.

2. This Index is released by the World Intellectual Property Organization (WIPO).

Which of the above statement(s) is/are true?

a. 1 only

b. 2 only

c. Both 1 and 2

d. Neither 1 nor 2

3) Consider the following statements

1. Methane is a non-flammable greenhouse gas.

2. It is less potent than carbon dioxide in terms of its global warming capacity.

3. Harit Dhara is an anti-methanogenic feed supplement for bovines and sheep.

Select the correct option:

a. 1 only

b. 1 and 2 only

c. 3 only

d. 2 and 3 only

4) Which of the following statements is incorrect about Black Tigers?

a. Black Tigers are result of true melanism, a condition characterised by unusually high deposition of melanin

b. Black Tigers have stripes that are broadened and fused together

c. Black Tigers are caused by recessive (hidden) genes, which are rare in nature

d. Natural selection is also favouring melanistic leopards

5) Coal Beneficiation is a process of?

a. Mining the coal

b. Processing coal to meet industrial requirements

c. Cleaning of coal, so that dust and other unwanted particles are removed

d. None of the above

Answers

1. d

Augmented Reality (AR) is an enhanced version of the real physical world that is achieved through the use of digital visual elements, sound, or other sensory stimuli delivered via technology.

Almost any person with a smart phone can get access to AR, making it more efficient than VR as a branding and gaming tool.

Virtual Reality (VR) produces an entirely computer-generated simulation of an alternate world.

These immersive simulations can create almost any visual or place imaginable for the player using special equipment such as computers, sensors, headsets, and gloves.

Merged Reality or Mixed Reality is when the real and virtual world are merged and objects from both can interact.

For example, Intel‘s Project Alloy uses mixed reality to allow people to use their real hands in the virtual world.

Before this point, users had to hold controllers that represented their hands.

2. c

Global Innovation Index 2021

World Intellectual Property Organization (WIPO) has released the Global Innovation Index (GII) 2021.

India has climbed 2 spots and has been ranked 46th. India‘s rank has been consistently rising in the last few years from 81 (2015) to 46 (2021).

The GII ranks 131 countries and economies based on their innovation capabilities, including roughly 80 indicators, grouped into innovation inputs and outputs.

3. c

Page 49: INDEX [iasparliament.s3.ap-south-1.amazonaws.com]

50

www.shankariasacademy.com | www.iasparliament.com

The US President has announced the Global Methane Pledge, which is an US-EU led effort to cut methane emissions by a third by the end of this decade.

Methane is the simplest hydrocarbon, consisting of one carbon atom and four hydrogen atoms (CH4).

Methane is a powerful greenhouse gas, which is flammable and is used as a fuel worldwide.

Around 60% of methane comes from human-influenced sources, and 40% is emitted from natural sources.

Harit Dhara is an anti-methanogenic feed supplement developed by an Indian Council of Agricultural Research (ICAR) institute - National Institute of Animal Nutrition and Physiology (NIANP), Bengaluru.

When given to bovines and sheep, it cuts down their methane emissions by 17-20%, and also results in higher milk production and body weight gain.

4. a

5. b

Coal Beneficiation is a process of processing coal to meet industrial requirements.

22-09-2021

1) Consider the following statements with respect to Strengthening the Potential of India (SPIN) Scheme

1. It is a specially-designed no-subsidy program that enables to make potters self-sustainable.

2. The Scheme enables the registered potters to get easy direct loans from the banks under Pradhan Mantri Shishu Mudra Yojana.

Which of the statement(s) given above is/are correct?

a. 1 only

b. 2 only

c. Both 1 and 2

d. Neither 1 nor 2

2) Which of the following are not the criteria for a celestial object to be declared Planet by the International Astronomical Union?

1. It must orbit the Sun.

2. It should be free floating.

3. It should be massive enough to acquire an approximately spherical shape.

4. The mass or minimum mass is equal to or less than 30 Jupiter masses.

5. It has to clear its orbit by exerting maximum gravitational pull within its orbit.

Select the correct answer using the codes given below:

a. 2 and 4 only

b. 2 and 3 only

c. 3 and 5 only

d. 1 and 4 only

3) Consider the following statements with respect to Coffee Act, 1942

1. It constituted the Coffee Board of India, a statutory organization under the Ministry of Commerce & Industry.

2. The Act states that Coffee cannot be exported from India by anyone except the Board or with the authorization of the Board.

Which of the statement(s) given above is/are correct?

a. 1 only

b. 2 only

c. Both 1 and 2

d. Neither 1 nor 2

4) Consider the following statements with respect to R-value

1. It indicates the speed at which a disease is spreading in the country, along with the efficiency of the spread.

2. Higher the R-value number, the more contagious is the disease caused by the virus and the faster it will spread in the community.

Which of the statement(s) given above is/are correct?

a. 1 only

b. 2 only

c. Both 1 and 2

d. Neither 1 nor 2

Page 50: INDEX [iasparliament.s3.ap-south-1.amazonaws.com]

51

www.shankariasacademy.com | www.iasparliament.com

5) Which of the following does not qualify as criminal contempt of the Supreme Court?

a. Wilful disobedience to any judgements of the Court.

b. Scandalising or lowering the authority of the Court.

c. Interference with the due course of a judicial proceeding.

d. Intimidating a witness

Answers

1. c

Khadi and Village Industries Commission (KVIC) launched the Strengthening the Potential of India (SPIN) Scheme and set up a pottery cluster under SFURTI Scheme in Varanasi to empower the marginalized potters.

SPIN Scheme

SPIN Scheme is a specially-designed no-subsidy program that enables to make potters self-sustainable.

It enables the registered potters to get easy direct loans from the banks under Pradhan Mantri Shishu Mudra Yojana.

Under the SPIN Scheme, KVIC is acting as a facilitator for financial aid to potters through RBL bank and also providing training to the artisans.

There will be no financial burden on the exchequer and the loan will be repaid by the potter in easy instalments.

2. a

Planet Nine

A study says that it has found more evidence for a distant giant planet in our solar system called Planet Nine.

It has found that the Planet Nine could have a mass of 6.2 (+2.2/-1.3) Earth masses. It may be a typical extra-solar super-Earth.

Planet Nine is not a new discovery. It had appeared in the images taken by the Hubble Space Telescope in 2004.

It orbits far beyond Neptune in a highly eccentric orbit around the Sun.

A 2018 study noted that a trans-Neptunian object called 2015 BP519 had an unusual trajectory as it was affected by Planet Nine‘s strong gravity.

Criteria for Planet

International Astronaumical Union determines the criteria and vote on the decisions.

The 3 conditions for a celestial object to be declared as a planet

1. It must orbit the Sun.

2. It should be massive enough to acquire an approximately spherical shape.

3. It has to clear its orbit (being the object that exerts maximum gravitational pull within its orbit).

Dwarf Planet - If a celestial object satisfies any 2 of the above 3 conditions, it is called as dwarf planet.

It orbits the sun, and has enough mass to assume a nearly round shape.

It has not cleared the neighbourhood around its orbit and is not a moon.

Exoplanet is a planet outside the Solar System. The first confirmation of detection of exoplanet occurred in 1992.

3. c

Simplification of Coffee Act

Ministry of Commerce & Industry decided to simplify the Coffee Act, 1942.

The Act states that Coffee cannot be exported from India by anyone except the Board or with the authorization of the Board.

Coffee Board of India

It is a statutory organization constituted under the Coffee Act, 1942.

It functions under the Ministry of Commerce & Industry.

4. c

R Value

Page 51: INDEX [iasparliament.s3.ap-south-1.amazonaws.com]

52

www.shankariasacademy.com | www.iasparliament.com

The R value, which reflects how rapidly the COVID-19 pandemic is spreading, dropped to 0.92 by mid-September after going over one in August-end.

R-value

R-value or R-factor or R0 (R-naught) or Reproduction Rate refers to how many persons an infected person infects on an average.

It indicates the speed at which the Covid infection is spreading in the country, along with the ‗efficiency‘ of the spread.

Higher the R-value number, the more contagious is the disease caused by the virus and the faster it will spread in the community.

5. a

Wilful disobedience to any judgements of the Court is considered a civil contempt.

23-09-2021

1) Consider the following statements with respect to Space weather

1. It refers to the conditions in the solar wind and near-Earth space, which can adversely affect the performance of space-borne and ground-based technological systems.

2. The space weather near the Earth is mainly due to Coronal Mass Ejections (CMEs), which are frequent explosive expulsions of huge magnetized plasma from the Sun.

3. The geomagnetic storm, a perturbation in the Earth‘s magnetic field, is an example of space weather event.

Which of the statement(s) given above is/are correct?

a. 1 and 2 only

b. 1 and 3 only

c. 2 and 3 only

d. 1, 2 and 3

2) Which of the following are the objectives of Beach Environment & Aesthetic Management Services (BEAMS) Program?

1. To abate pollution in coastal waters & beaches

2. To promote sustainable development of beach amenities/ facilities

3. To protect & conserve coastal ecosystems & natural resources

Select the correct answer using the codes given below:

a. 2 only

b. 1 and 2 only

c. 2 and 3 only

d. 1, 2 and 3

3) Consider the following statements with respect to Greater One-horned Rhinoceros

1. Currently, it has been found in India, Pakistan and parts of Nepal, Bangladesh and Bhutan.

2. Its populations are increasing overall due to strict protection and habitat management in both India and Nepal.

3. The New Delhi Declaration on Asian Rhinos, 2019 was signed between India, Bhutan, Nepal, Indonesia and Malaysia.

Which of the statement(s) given above is/are correct?

a. 2 only

b. 1 and 3 only

c. 2 and 3 only

d. 1, 2 and 3

4) Vishnuonyx, sometimes seen in the news recently, refers to?

a. Genus of otters lived in the major rivers of southern Asia

b. A family of open source Unix-like operating system

c. AI-assisted, satellite-operated robotic machine gun

d. None of the above

5) Which of the following organizations has published the State Food Safety Index (SFSI) recently?

a. UN World Food Programme

b. Food and Agricultural Organization

Page 52: INDEX [iasparliament.s3.ap-south-1.amazonaws.com]

53

www.shankariasacademy.com | www.iasparliament.com

c. International Food Policy Research Institute

d. Food Safety and Standards Authority of India

Answers

1. d

A recent study has shown how conditions and events in the solar atmosphere like coronal mass ejections influence the accuracy of space weather prediction, which is crucial for the health of our satellites.

This understanding will aid the interpretation of data from the upcoming Aditya-L1, India's first solar mission.

Space weather

It refers to the conditions in the solar wind and near-Earth space, which can adversely affect the performance of space-borne and ground-based technological systems.

The space weather near the Earth is mainly due to Coronal Mass Ejections (CMEs), which are frequent explosive expulsions of huge magnetized plasma from the Sun into its surroundings, which can blow past the Earth.

Example of space weather events is the geomagnetic storm, a perturbation in the Earth‘s magnetic field, which can last for few hours to few days.

Understanding of how events in the solar atmosphere influence space weather is necessary for monitoring and maintaining our satellites.

2. d

Foundation for Environment Education (FEE) in Denmark which accords the globally recognized eco-label – ―Blue Flag‖ certification has accorded the Blue Flag Certification for 2 new beaches this year –Kovalam in Tamil Nadu and Eden in Puducherry beaches.

With this, India now has 10 Blue Flag beaches

Blue Flag Certification

Society of Integrated Coastal Management (SICOM) in its pursuit of Sustainable Development of the coastal regions of India

embarked upon a highly acclaimed & flagship program ―BEAMS‖ (Beach Environment & Aesthetics Management Services).

This was aimed at striving for the coveted International eco-label "Blue flag‖, accorded by The Foundation of Environment Education, FEE Denmark.

Beach Environment & Aesthetics Management Services (BEAMS)

This program promotes beach tourism and recreation in absolute harmony with nature

The objective of the BEAMS program is

1. to abate pollution in coastal waters & beaches

2. to promote sustainable development of beach amenities/ facilities

3. to protect & conserve coastal ecosystems & natural resources

4. to encourage local authorities & stakeholders to strive and maintain high standards of cleanliness, hygiene, safety and security for beachgoers in accordance with coastal environment norms & regulations

Page 53: INDEX [iasparliament.s3.ap-south-1.amazonaws.com]

54

www.shankariasacademy.com | www.iasparliament.com

3. c

Assam will mark World Rhino Day — September 22 — with a special ceremony by burning a stockpile of nearly 2,500 horns of the one-horned rhinoceros.

Greater one-horned Rhinoceros

Habitat - Historically, the Greater One-horned Rhinoceros were found across the entire northern part of the Indian subcontinent, along the Indus, Ganges, and Brahmaputra River basins, from Pakistan to the Indian-Burmese border, including parts of Nepal, Bangladesh and Bhutan.

Currently, the Greater One-horned Rhinoceros is found in eight protected areas in India (Kaziranga, Pabitora, Manas, Orang, Jaldapara, Gorumara, Dudhwa, Katerniaghat) and in four protected areas in Nepal (Chitwan, Bardia, Suklaphanta, Parsa).

In India, rhinoceros from Gorumara National Park frequently visit adjoining forests; Nepali rhinos frequently visit adjoining areas in India.

New Delhi Declaration on Asian Rhinos, 2019 - India has signed the New Delhi Declaration on Asian Rhinos, 2019 with Bhutan, Nepal, Indonesia and Malaysia to increase the population of three species of Asian Rhinos namely Greater one-horned rhinoceros, Javan rhino and Sumatran rhino.

Indian Rhino Vision (IRV-2020) - Government of Assam has launched the Indian Rhino Vision (IRV-2020) in partnership with WWF India, International Rhino Foundation, and several other organizations.

4. a

Vishnuonyx

Recently, the fossil of Vishnuonyx neptuni (Neptune‘s Vishnu) species was found in the Hammerschmiede area, a fossil site in Bavaria, Germany.

Between 12.5 and 14 million years ago, the members of the Vishnuonyx genus of otters lived in the major rivers of southern Asia.

Fossils of these now extinct otters were first discovered in sediments found in the foothills of the Himalayas.

This is the first discovery of any member of the Vishnuonyx genus in Europe; it is also its most northern and western record till date.

Vishnuonyx were mid-sized predators that weighed 10-15 kg.

They depended on water and could not travel long distances over land.

Its travels over 6,000 km to Europe were probably made possible by the geography of 12 million years ago, when the Alps were recently formed.

These Alps and the Iranian Elbrus Mountains were separated by an ocean basin, which would have made it easier for the otters to cross it.

5. d

State Food Safety Index

Recently, the Food Safety and Standards Authority of India (FSSAI) released the 3rd State Food Safety Index (SFSI).

This Index measures the performance of States across 5 parameters of food safety, which include,

1. Human Resources and Institutional Arrangements,

2. Compliance,

3. Food Testing - Infrastructure and Surveillance,

4. Training and Capacity Building and

5. Consumer Empowerment.

It is a dynamic quantitative and qualitative benchmarking model that provides an objective framework for evaluating food safety across all States/UTs.

Among the larger states, this year, Gujarat, Kerala and Tamil Nadu secured top ranks.

Among the smaller states, Goa, Meghalaya and Manipur secured top ranks.

Among UTs, Jammu & Kashmir, Andaman & Nicobar Islands and New Delhi secured top ranks.

Page 54: INDEX [iasparliament.s3.ap-south-1.amazonaws.com]

55

www.shankariasacademy.com | www.iasparliament.com

24-09-2021

1) DRINK, often seen in the news recently, refers to?

a. Alcohol Museum

b. Banking malware

c. Campaign on Climate Change

d. Clean water & Sanitation Campaign

2) With respect to Freedom on the Net Index, consider the following statements:

1. It is an annual report which deals with the state of Internet freedom across countries.

2. It will be published by the U.S. based not-for-profit entity Freedom House.

Which of the statement(s) given above is/are correct?

a. 1 only

b. 2 only

c. Both 1 and 2

d. Neither 1 nor 2

3) Consider the following statements with respect to WHO Air quality guidelines

1. It recommends levels and interim targets for common air pollutants like PM, O3, NO2, and SO2.

2. The guidelines will be reviewed and updated annually by the World Health Organization (WHO).

3. These guidelines are neither standards nor legally binding.

Which of the statement(s) given above is/are correct?

a. 1 and 2 only

b. 1 and 3 only

c. 2 and 3 only

d. 1, 2 and 3

4) With respect to Fast and Secured Transmission of Electronic Records (FASTER) System, consider the following statements:

1. Under the system, courts can send e-authenticated copies of bail orders and stay

orders, through secured electronic communication to duty officers at prisons.

2. It was introduced by the Supreme Court of India.

Which of the statement(s) given above is/are correct?

a. 1 only

b. 2 only

c. Both 1 and 2

d. Neither 1 nor 2

5) Consider the following statements:

1. India is not having common law for both civil and criminal matters.

2. The Principle of Natural Justice concerns procedural fairness and ensures a fair decision is reached by an objective decision maker.

Which of the statements given above is/are correct?

a. 1 only

b. 2 only

c. Both 1 and 2

d. Neither 1 nor 2

Answers

1. b

DRINK Malware

India's Computer Emergency Response Team (CERT-In) has issued an advisory to bank customers of an android malware called, Drink, that steals information and money.

This new banking malware has evolved from a primitive SMS stealer in 2016 to a banking trojan that persuades users to enter sensitive banking information.

The attack campaign can effectively jeopardise security of sensitive customer data and lead to large scale attacks and financial fraud.

2. c

Freedom on the Net Report

U.S based not-for-profit organization, Freedom House has published a report titled "Freedom on the Net 2021: The Global Drive to Control Big Tech" recently.

Page 55: INDEX [iasparliament.s3.ap-south-1.amazonaws.com]

56

www.shankariasacademy.com | www.iasparliament.com

Freedom on the Net is an annual report providing analytical reports and numerical ratings regarding the state of Internet freedom for countries worldwide.

The report for the year 2021 claimed that the global internet freedom declined for the 11th consecutive year.

China and Pakistan are among the bottom 10 countries on the global rankings on internet freedom released by an international freedom advocacy group.

Myanmar, Belarus, and Uganda witnessed the greatest deteriorations in internet freedom amid electoral and constitutional crises.

On the other hand, Iceland, Canada and Germany were among the top 10 countries with greater internet freedom.

Countries with scores between 70 to 100 are designated as ‗Free‘ on internet freedom status while those with points between 40 to 69 are designated as ‗Partly-free‘. Countries scoring less than 39 are designated as ‗Not free‘.

3. b

WHO’s Air quality guidelines (AQG)

Since 1987, the world body has periodically issued health-based air quality guidelines to assist governments and civil society to reduce human exposure to air pollution and its adverse effects.

The WHO has updated the global air pollution standards recently, a first since 2005. (not updated annually)

The World Health Organization‘s Air quality guidelines (AQG) serve as a global target for national, regional and city governments to work towards improving their citizen‘s health by reducing air pollution.

The WHO guidelines also propose interim targets that can be met en route to achieving the final target.

The WHO Air quality guidelines recommend levels and interim targets for common air pollutants: PM, O3, NO2, and SO2.

Although the guidelines are neither standards nor legally binding criteria, they are designed to offer guidance in reducing the health

impacts of air pollution based on expert evaluation of current scientific evidence.

Air Quality Standards in India

India's National Ambient Air Quality standards — last revised in 2009 — specify an annual limit of 60 microgram per cubic metre for PM10 and 100 for a 24 hour period.

Similarly it's 40 for PM2.5 annually and 60 on a 24-hour period.

India is thus far from meeting even the existing guidelines.

As part of improving air quality standards, India has ongoing a National Clean Air Programme that aims for a 20% to 30% reduction in PM concentrations by 2024 in 124 cities, keeping 2017 as the base year for the comparison of concentration.

Page 56: INDEX [iasparliament.s3.ap-south-1.amazonaws.com]

57

www.shankariasacademy.com | www.iasparliament.com

4. c

FASTER System

The Supreme Court introduced the Fast and Secured Transmission of Electronic Records (FASTER) system.

This system will help in communicating crucial decisions including orders on bail and stay of arrest electronically to prison authorities and investigating agencies through a secure channel.

The system is meant to ensure that undertrials aren‘t made to wait for days on end behind bars to be released because the certified hard copies of their bail orders took time to reach the prison.

It would also prevent unnecessary arrests and custody of people even after the court had already granted them its protection.

It may even communicate a stay on an execution ordered by the final court on time.

The FASTER system will be a big fillip to the fundamental rights of life, dignity and personal liberty.

5. b

Common Law

Common law (also known as case law or precedent) is law developed by judges, courts, and similar tribunals, stated in decisions that nominally decide individual cases but that in addition has precedential effect on future cases.

Common law can be seen in contrast to statutes which are adopted through the legislative process, and regulations which are promulgated by the executive branch.

India, being a common law country, derives most of its modern judicial framework from the British legal system.

There exists a uniform system of justice dispensation, with the Supreme Court at the apex and High Courts in the States (provincial units in India), as well as numerous other subordinate courts.

25-09-2021

1) With respect to the Ocean State Report – 5, released recently, consider the following statements:

1. It was released by the International Maritime Organization (IMO).

2. According to the report, sea ice is steadily decreasing in the Arctic.

3. The warming ocean waters have caused many marine species to move towards cooler waters.

Which of the statement(s) given above is/are correct?

a. 1 and 2 only

b. 1 and 3 only

c. 2 and 3 only

d. 1, 2 and 3

2) With respect to PLI Scheme for Drones, consider the following statements:

1. Under the scheme, incentives for a manufacturer of drones and components shall be as high as 20% of the value addition made.

2. The scheme is also available to developers of drone-related IT products.

3. Incentives given under the scheme will be kept at a constant PLI rate whereas in other sectors, it would be reduced every year.

Which of the statements given above are correct?

a. 1 and 2 only

b. 1 and 3 only

c. 2 and 3 only

d. 1, 2 and 3

3) Which of the following countries are part of QUAD, AUKUS and Five Eyes Alliance?

1. Japan

2. Australia

3. United Kingdom

4. United States of America

Select the correct answer using the codes given below:

a. 1 and 2 only

Page 57: INDEX [iasparliament.s3.ap-south-1.amazonaws.com]

58

www.shankariasacademy.com | www.iasparliament.com

b. 2 and 4 only

c. 1, 2 and 4 only

d. 1, 2, 3 and 4

4) Which of the following countries has recently declared that all crypto-currency transactions as illegal?

a. Iran

b. China

c. Saudi Arabia

d. New Zealand

5) With respect to Bad Banks, consider the following statements:

1. It takes over the bad loans of commercial banks, manages them and finally recovers the money over a period of time.

2. A bad bank should not be involved in lending and taking deposits.

3. The concept of bad bank is an Indian innovation with the establishment of National Asset Reconstruction Company Limited.

Which of the statement(s) given above is/are correct?

a. 2 only

b. 1 and 2 only

c. 1 and 3 only

d. 1, 2 and 3

Answers

1. c

Ocean State Report 5

The Copernicus Marine Environmental Monitoring Service has released the 5th edition of the Ocean State Report (OSR 5).

The report has said that the global ocean is undergoing severe changes from natural variations, over-exploitation and anthropogenic influences.

These changes caused the sea level to rise by 3.1 millimeters each year on an average between January 1993 and May 2020.

The report showed that the sea ice is steadily decreasing in the Arctic.

It has also showed that the warming ocean waters have caused many marine species to move towards cooler waters.

This migration has led to the introduction of non-native and invasive species to different marine ecosystems.

Around 50% of Earth‘s oxygen production takes place in the ocean, sustaining marine life cycles.

This is threatened by growing human activities leading to climate change and eutrophication.

This deoxygenates the oceans and seas and has adverse consequences on the marine life.

2. d

The PLI Scheme for Drones (approved on 15 Sep 2021)

Budget - The total amount allocated for the PLI scheme for drones and drone components is INR 120 crore spread over three financial years.

This amount is nearly double the combined turnover of all domestic drone manufacturers in FY 2020-21.

Incentives - The incentive for a manufacturer of drones and drone components shall be as high as 20% of the value addition made by her.

Exceptions - The Government has agreed to keep the PLI rate constant at 20% for all three years, an exceptional treatment given only to the drone industry.

In PLI schemes for other sectors, the PLI rate reduces every year.

The Government has agreed to fix the minimum value addition norm at 40% of net sales for drones and drone components instead of 50%, another exceptional treatment given to the drone industry.

This will allow widening the number of beneficiaries.

Eligibility - The Government has kept the eligibility norm for MSME and startups in terms of annual sales turnover at a nominal level - INR 2 cr (for drones) and INR 50 lakhs

Page 58: INDEX [iasparliament.s3.ap-south-1.amazonaws.com]

59

www.shankariasacademy.com | www.iasparliament.com

(for drone components). This will allow widening the number of beneficiaries.

Eligibility norm for non-MSME companies in terms of annual sales turnover has been kept at INR 4 crore (for drones) and INR 1 crore (for drone components).

Coverage - The Government has agreed to widen the coverage of the incentive scheme to include developers of drone-related IT products also.

3. b

4. b

China‘s central bank has recently said that all financial transactions involving cryptocurrencies are illegal.

It also bans all related financial activities involving cryptocurrencies, such as trading crypto, selling tokens, transactions involving virtual currency derivatives and illegal fundraising.

Trading crypto-currency has officially been banned in China since 2019, but has continued online through foreign exchanges.

Mining Migration

The technology at the core of many crypto-currencies, including Bitcoin, relies on many distributed computers verifying and checking transactions on a giant shared ledger known as the blockchain.

As a reward, new "coins" are randomly awarded to those who take part in this work - known as crypto "mining".

China, with its relatively low electricity costs and cheaper computer hardware, has long been one of the world's main centres for mining.

The activity is so popular there that gamers have sometimes blamed the industry for a global shortage of powerful graphics cards, which miners use for processing crypto-currencies.

The Chinese crackdown has already hit the mining industry.

In September 2019, China accounted for 75% of the world's Bitcoin energy use. By April 2021, that had fallen to 46%.

5. b

The Union Cabinet has recently approved a 30,600 crore rupees backstop facility for guaranteeing securities to be issued by the National Asset Reconstruction Company Ltd. (NARCL).

NARCL is the bad bank of India that is being set up to help aggregate and consolidate lenders‘ non-performing assets (NPAs) or bad loans.

Bad Banks

Technically, a bad bank is an asset reconstruction company (ARC) or an asset management company that takes over the bad loans of commercial banks, manages them and finally recovers the money over a period of time.

The bad bank is not involved in lending and taking deposits.

Page 59: INDEX [iasparliament.s3.ap-south-1.amazonaws.com]

60

www.shankariasacademy.com | www.iasparliament.com

It just helps commercial banks clean up their balance sheets and resolve bad loans.

The takeover of bad loans is normally below the book value of the loan and the bad bank tries to recover as much as possible subsequently.

US-based Mellon Bank created the first bad bank in 1988.

27-09-2021

1) Consider the following pairs:

Coir Products Used to Save

1. Coir pith - Water

2. Coir Geotextiles - Soil

3. Coir Wood - Trees and forest

Which of the pair(s) given above is/are correctly matched?

a. 3 only

b. 1 and 2 only

c. 1 and 3 only

d. 1, 2 and 3

2) Arrange the following Lakes of Kerala from North to South

1. Vembanadu

2. Kayamkulam

3. Asthamudi

Select the correct answer using the code given below:

a. 2 - 1 - 3

b. 1 - 2 - 3

c. 3 - 2 - 1

d. 3 - 1 – 2

3) Which one of the following statements with respect to Atal Bimit Vyakti Kalyan Yojana is correct?

a. It is a scheme launched by Ministry of Personnel, Public Grievances and Pensions.

b. Under this, Insured Persons will be given a relief after retirement amounting to 25% of the average per day earning.

c. The relief will be paid up to a maximum of 90 days of unemployment, as a once-in-lifetime measure.

d. The relief shall be paid through e-Vouchers, which will be delivered directly to the mobile of the beneficiaries

4) Consider the following statements with respect to Judima, often seen in the news recently

1. It is a rice wine made from sticky rice, which is steamed and mixed with traditional herbs.

2. The wine is brewed by the Dimasa tribal community of Assam and is traditionally made by women.

Which of the statement(s) given above is/are correct?

a. 1 only

b. 2 only

c. Both 1 and 2

d. Neither 1 nor 2

5) Consider the following statements with respect to LiFi technology

1. It is a wireless communication technology that uses light rather than radio frequencies to transmit data.

2. LiFi can be up to 100 times faster than Wi-Fi.

Which of the statement(s) given above is/are correct?

a. 1 only

b. 2 only

c. Both 1 and 2

d. Neither 1 nor 2

Answers

1. d

Coir Products are eco friendly in nature and gained ―Eco Mark‖ certification by the Ministry of Environment and Forests.

Page 60: INDEX [iasparliament.s3.ap-south-1.amazonaws.com]

61

www.shankariasacademy.com | www.iasparliament.com

Coir products save environment and help to reverse Global Warming.

Coir Pith will be used as a valuable bio- fertiliser, soil conditioner and can be used to save water

Coir Geotextiles are used to prevent soil erosion and save soils

Coir Woods are used to save trees and forest.

2. b

3. c

Atal Bimit Vyakti Kalyan Yojana (ABVKY)

It is a scheme launched by ESI Corporation.

Under this, if the Insured Person (IP) loses his/her job, he/she is given a relief amounting to 25% of the average per day earning during the previous four contribution periods (total earning during the four contribution periods/730).

The relief will be paid up to a maximum of 90 days of unemployment, as a once-in-lifetime measure.

The Relief under ABVKY shall be paid/ payable by Branch Office to IPs directly in their bank account only, upon submission of the claim.

4. c

Judima

Judima has become the first beverage from the north-eastern region to earn the geographical indication (GI) tag.

Judima is a rice wine made from sticky rice, which is steamed and mixed with traditional herbs.

The wine is brewed by the Dimasa tribal community in Assam. It is traditionally made by women.

The wine has a distinct sweet taste and takes around one week to prepare and it can be stored for years.

5. c

Students' Educational and Cultural Movement of Ladakh (SECMOL), India's highest altitude educational organisation, has become the 1st institute in the Union Territory to have an internet connection using LiFi technology.

LiFi Network

Light Fidelity (LiFi) is a wireless communication technology that transmits data with a light beam spectrum through open space in outdoor and indoor environments.

It is a Visible Light Communications (VLC) system that has,

1. Solid-state lighting (SSL) such as LED bulbs,

2. Photo-detector (photodiode) to receive light signals and

Page 61: INDEX [iasparliament.s3.ap-south-1.amazonaws.com]

62

www.shankariasacademy.com | www.iasparliament.com

3. Signal processing element to convert the data (embedded in the light beam) into ‗stream-able‘ content.

Data is fed into an LED light bulb (with signal processing technology). Then the LED bulb sends data at rapid speeds to the photo-detector.

Uses - LiFi systems provide ultra-fast, safe and cheap data connections.

They are chiefly useful in urban areas where radio spectra are congested.

They are also very useful in rural areas wherein Fiber Optic Cables or networks are not reachable.

Specialty - LiFi can be up to 100 times faster than Wi-Fi.

28-09-2021

1) With respect to All-India Quarterly Establishment based Employment Survey (AQEES), consider the following statements:

1. The report will provide quarterly updates on employment in both organised and unorganised segments.

2. The report will be based on nine selected sectors which account for a majority of total employment in the non-farm establishments.

3. The report will be prepared by the Labour Bureau.

Which of the statements given above are correct?

a. 1 and 2 only

b. 1 and 3 only

c. 2 and 3 only

d. 1, 2 and 3

2) Which one of the following is the objective of XENON1T Experiment, sometimes seen in the news recently?

a. To search for Dark Matter

b. To search for Extra Terrestrial Life

c. To search for Gravitational Waves

d. None of the above

3) Consider the following statements with respect to Akash Missile

1. It is a part of five major missile developed under the Integrated Guided Missile Development Programme (IGMDP).

2. It is primarily a Short Range Surface to Air Missile built to provide air defence cover to the vulnerable areas.

Which of the statement(s) given above is/are correct?

a. 1 only

b. 2 only

c. Both 1 and 2

d. Neither 1 nor 2

4) Consider the following statements with respect to National Disaster Management Authority (NDMA)

1. NDMA is not a statutory body and derives its powers from the Disaster Management Act, 2005.

2. It is the apex body for Disaster Management in India, headed by the Union Defence Minister.

3. Setting up of National, State and District level disaster management authority is mandated by the Disaster Management Act, 2005.

Which of the statement(s) given above is/are not correct?

a. 3 only

b. 1 and 2 only

c. 1 and 3 only

d. 1, 2 and 3

5) How is Dark Matter different from Dark Energy?

1. Dark energy attracts and holds galaxies together whereas dark matter repels and causes the expansion of our universe.

2. Despite both components being invisible, dark energy is even more elusive than dark matter.

Which of the statement(s) given above is/are correct?

a. 1 only

b. 2 only

c. Both 1 and 2

Page 62: INDEX [iasparliament.s3.ap-south-1.amazonaws.com]

63

www.shankariasacademy.com | www.iasparliament.com

d. Neither 1 nor 2

Answers

1. d

Union Minister for Labour and Employment has recently released the report of first quarter of Quarterly Employment Survey (QES) from April to June 2021.

It is a part of All-India Quarterly Establishment-based Employment Survey (AQEES).

All-India Quarterly Establishment-based Employment Survey (AQEES)

The AQEES has been taken to provide quarterly updates about the employment and related variables of establishments, in both organised and unorganised segments of nine selected sectors.

These sectors altogether account for a majority of the total employment in the non-farm establishments.

These nine selected sectors are Manufacturing, Construction, Trade, Transport, Education, Health, Accommodation and Restaurant, IT/ BPO and Financial Services.

About AQEES

The AQEES will be prepared by the Labour Bureau.

There are two components under AQEES, Quarterly Employment Survey (QES) and Area Frame Establishment Survey (AFES).

The Quarterly Employment Survey (QES) under AQEES would provide the employment estimates for the establishments employing 10 or more workers. The Area Frame Establishment Survey (AFES) would provide the employment estimates for the establishments recruiting 9 or less workers.

2. a

Dark energy, the mysterious form of energy that makes up about 68% of the universe, has intrigued physicists and astronomers for decades.

With advanced technologies and newer experiments, scientists have found certain clues about it and an international team of researchers made the first putative direct detection of dark energy.

XENON 1T Experiment

The XENON1T experiment is the world‘s most sensitive dark matter experiment and was operated deep underground at the INFN Laboratori Nazionali del Gran Sasso in Italy.

It is a 3500kg liquid xenon detector to search for the elusive Dark Matter.

The finding also suggests that experiments like XENON1T, which are designed to detect dark matter, could also be used to detect dark energy.

3. c

The Defence Research and Development Organisation (DRDO) has recently conducted the maiden flight test of the new version of Akash Missile — Akash Prime.

This comes after the maiden test of another Akash version, Akash-NG (New Generation), in January, 2021.

Origin of AKASH

The development of the Akash SAM was started by the DRDO in the late 1980s as part of the Integrated Guided Missile Development Programme.

Akash is primarily a Short Range Surface to Air Missile built to provide air defence cover to the vulnerable areas.

The initial version of the Akash has an operational range of 27-30 km and a flight altitude of around 18 km.

According to the Ministry of Defence, the Akash Missile system is 96 per cent indigenised, one of the highest proportions of the indigenisation.

Advanced Version of AKASH

AKASH Prime

The Akash Prime has the same range as that of the earlier version but has a crucial new addition — that of an indigenous active Radio

Page 63: INDEX [iasparliament.s3.ap-south-1.amazonaws.com]

64

www.shankariasacademy.com | www.iasparliament.com

Frequency (RF) seeker for improved accuracy to hit aerial targets.

Other improvements in the system ensure more reliable performance under low temperature environments at higher altitudes.

These new additions have been to provide air defence cover for vital installations and sensitive areas in high-altitude regions.

AKASH New Generation (NG)

It is a new generation SAM, primarily designed for the IAF with an aim of intercepting high maneuvering aerial threats that have low Radar Cross Section (RCS), which is the electromagnetic signature of the object.

The NG version has an extended range of up 70 km, is sleeker, lighter and has much smaller ground system footprint.

As an additional feature, Akash NG is canisterised, which means that it is stored and operated from specially designed compartments.

In the canister, the inside environment is controlled thus along with making its transport and storage easier, the shelf life of weapons also improves significantly.

4. b

17th Formation Day of the National Disaster Management Authority (NDMA) was celebrated recently in New Delhi.

The theme of this year's Formation Day is Cascading effects of disaster events in the Himalayan region.

National Disaster Management Authority (NDMA)

It is the apex body for Disaster Management in India.

It is a statutory body established through the Disaster Management Act, 2005.

It will be headed by the Prime Minister of India.

Setting up of NDMA and the creation of an enabling environment for institutional mechanisms at the State and District levels is mandated by the Disaster Management Act, 2005.

NDMA is mandated to lay down the policies, plans and guidelines for Disaster Management.

5. b

Everything we see, the planets, moons, massive galaxies, makes up less than 5% of the universe.

About 27% is dark matter and 68% is dark energy.

Dark energy Vs Dark matter

While dark matter attracts and holds galaxies together, dark energy repels and causes the expansion of our universe.

Despite both components being invisible, we know a lot more about dark matter, since its existence was suggested as early as the 1920s, while dark energy wasn‘t discovered until 1998.

Large-scale experiments like XENON1T have been designed to directly detect dark matter, by searching for signs of dark matter ‗hitting‘ ordinary matter, but dark energy is even more elusive.

29-09-2021

1) Consider the following statements with respect to Green Voyage 2050 Project

1. It is a partnership project between the Government of Norway and International Maritime Organization (IMO).

2. It supports developing countries, including Small Islands Developing States and Least Developed Countries in their efforts to reduce GHG emissions from ships.

3. India is one of the partnering countries of the project.

Which of the statements given above are correct?

a. 1 and 2 only

b. 1 and 3 only

c. 2 and 3 only

d. 1, 2 and 3

2) Which of the following actions does not qualify for disqualifications on ground of defection?

Page 64: INDEX [iasparliament.s3.ap-south-1.amazonaws.com]

65

www.shankariasacademy.com | www.iasparliament.com

a. If a member elected on the ticket of a party voluntarily gives up membership of the party or votes in the House against the party's wishes

b. When a legislator who has won his or her seat as an independent candidate joins a political party after the election

c. If a nominated member joins a party during the first six months of his/her nomination

d. If a member goes out of his/her party as a result of a merger of the party with another party

3) Consider the following statements with respect to Bhagat Singh

1. He coined the term Inquilab Zindabad and raised it after bombing Central legislative Assembly Delhi in April, 1929.

2. He was tried and executed for shooting James Scott who was responsible for the death of Lala Lajpat Rai.

Which of the statement(s) given above is/are correct?

a. 1 only

b. 2 only

c. Both 1 and 2

d. Neither 1 nor 2

4) Consider the following statements with respect to Rabies

1. It is a zoonotic, viral disease spread to people from the saliva of infected animals.

2. It is one of the Neglected Tropical Diseases that predominantly affects poor and vulnerable populations of remote rural locations.

Which of the statement(s) given above is/are correct?

a. 1 only

b. 2 only

c. Both 1 and 2

d. Neither 1 nor 2

5) Consider the following statements with respect to Suvarnamukhi River

1. It is an east flowing river originated in the Chota Nagpur Plateau and flows through Jharkhand, West Bengal and Odisha.

2. It has no major tributaries and thus depends only on rainfall in its upper catchment.

Which of the statement(s) given above is/are correct?

a. 1 only

b. 2 only

c. Both 1 and 2

d. Neither 1 nor 2

Answers

1. d

Green Voyage 2050 Project

The Green Voyage 2050 Project is a partnership project between the Government of Norway and IMO launched in 2019 aiming to transform the shipping industry towards a lower carbon future.

It supports developing countries, including Small Islands Developing States (SIDS) and Least Developed Countries (LDCs), in meeting their commitment towards relevant climate change and energy efficiency goals, for international shipping, through supporting the Initial IMO GHG Strategy.

The International Maritime Organization is executing the GreenVoyage2050 Project through the Project Coordination Unit (PCU).

The Norwegian Ministry of Climate and Environment provides the funding for the GreenVoyage2050 Project.

Page 65: INDEX [iasparliament.s3.ap-south-1.amazonaws.com]

66

www.shankariasacademy.com | www.iasparliament.com

Partnering Countries

The GreenVoyage2050 Project is working with 12 countries: Azerbaijan, Belize, China, Cook Islands, Ecuador, Georgia, India, Kenya, Malaysia, Solomon Islands, South Africa, Sri Lanka.

The participating countries can be broadly categorized into ―New Pilot Countries‖ and ―Pioneer Pilot Countries‖.

New Pilot Countries (NPCs) are those which will, as a first step, undertake the development of a national maritime emissions assessment, establishing a baseline and build the information base, laying the foundation for targeted actions to reduce emissions from shipping.

Pioneer Pilot Countries (PPCs) are those which have already undertaken maritime emissions baseline work and have initiated development of their National Action Plan (NAP) to address GHG emissions from ships.

Background

GreenVoyage2050 builds upon two IMO projects that may be regarded as baseline projects in this area

1. GEF-UNDP-IMO GloMEEP Project (Global Maritime Energy Efficiency Partnerships)

2. EU-IMO GMN Project (Capacity Building for Climate Mitigation in the Maritime Shipping Industry)

2. c

The Tenth Schedule of the Constitution, popularly known as the anti-defection law, specifies the circumstances under which changing of political parties by legislators‘ invites action under the law.

The law covers three scenarios with respect to shifting of political parties by an MP or an MLA.

Three Scenarios

Elected Member - The first is when a member elected on the ticket of a political party ―voluntarily gives up‖ membership of such a party or votes in the House against the wishes of the party.

Independent Candidate - The second is when a legislator who has won his or her seat

as an independent candidate joins a political party after the election.

In both these instances, the legislator loses the seat in the legislature on changing (or joining) a party.

Nominated Member - The third scenario relates to nominated MPs. In their case, the law gives them six months to join a political party, after being nominated. If they join a party after such time, they stand to lose their seat in the House.

3. d

Bhagat Singh, Sukhdev and Rajguru were tried and then executed for fatally shooting a 21-year-old British police officer by the name of John Saunders, whom they had mistaken for British police superintendent James Scott, whom they had originally targeted.

The trio believed Scott was responsible for the death of the popular nationalist leader Lala Lajpat Rai, who succumbed to injuries sustained during a lathi charge.

While Singh, who had publicly announced avenging Rai‘s death, went into hiding for many months after this shootout, he resurfaced along with an associate Batukeshwar Dutt, and the two, in April 1929, set off two explosive devices inside the Central Legislative Assembly in Delhi, and then allowed themselves to be arrested, while shouting the famous slogan: ―Inquilab Zindabad―, or ―Long live the revolution‖.

The famous slogan Inquilab Zindabad was coined by the Urdu poet, Maulana Hasrat Mohani in 1921.

4. c

National Action Plan for dog Mediated Rabies Elimination by 2030

National Action Plan for dog Mediated Rabies Elimination by 2030 (NAPRE) was launched recently.

It wants to eliminate Dog mediated Rabies (Hadakwa disease) from India by 2030 through One Health Approach including community education, awareness programmes and vaccination campaigns.

Page 66: INDEX [iasparliament.s3.ap-south-1.amazonaws.com]

67

www.shankariasacademy.com | www.iasparliament.com

33% of global rabies deaths are recorded in India.

Rabies

Rabies is a zoonotic, viral disease spread to people from the saliva of infected animals.

Rabies is a 100% fatal but 100% vaccine preventable.

Rabies is a Neglected Tropical Diseases (NTD) that mostly affects poor and vulnerable populations. 80% of cases occur in rural areas.

Spread - Dogs are the main source of human rabies deaths, contributing up to 99% of all rabies transmissions to humans.

It is also transmitted through bats, coyotes, foxes, raccoons and skunks.

In rare cases, rabies can be spread when infected saliva gets into an open wound or the mucous membranes, such as the mouth or eyes.

Prevention - Interrupting transmission is feasible through vaccination of dogs and humans, and prevention of dog bites.

WHO leads the ‗United Against Rabies‘ to drive progress towards ―Zero human deaths from dog-mediated rabies by 2030‖.

5. b

Swarnamukhi River

The Swarnamukhi is an East Flowing river basin originated in the Eastern Ghats.

It rises at an elevation of 300 m in the Eastern Ghat ranges near Pakala village in Chittur district of Andhra Pradesh.

It runs generally in the North-Eastern direction passing through the famous Tirupati hills before joining into the Bay of Bengal.

This is an independent river and has no major tributaries and therefore its flow depends only on rainfall in its upper catchment.

30-09-2021

1) Which one of the following statements regarding PM POSHAN Scheme is incorrect?

a. It is a central sector scheme which aims to provide one hot cooked meal

b. The earlier name of the scheme was National Scheme for Mid Day Meal in Schools popularly known as Mid Day Meal Scheme

c. The scheme is proposed to be extended to students studying in pre-primary schools in addition to children from elementary classes

d. Social Audit of the scheme is made mandatory in all the districts

Page 67: INDEX [iasparliament.s3.ap-south-1.amazonaws.com]

68

www.shankariasacademy.com | www.iasparliament.com

2) SACRED Portal, sometimes seen in the news recently, refers to?

a. Employment Exchange for Elderly

b. Training portal for Health workers

c. Indian Temples Information Portal

d. National Database for Unorganised Workers

3) With respect to Trojan Asteroids, consider the following statements:

1. These are leftovers of the primordial material from which Jupiter and the other outer planets were formed.

2. It orbits the Sun in two loose groups, with one group leading ahead of Jupiter in its path and the other trailing behind the planet.

3. European Space Agency‘s Mission Lucy is the First Mission to study the Trojan Asteroids.

Which of the statements given above are correct?

a. 1 and 2 only

b. 1 and 3 only

c. 2 and 3 only

d. 1, 2 and 3

4) Ministry of Power has recently redesigned the existing Renewable Energy Certificate (REC) Mechanism. Which one of the following is not a feature proposed in revamped REC mechanism?

a. The Renewable Energy Certificate would be valid till it is sold

b. Central Electricity Regulatory Commission is responsible to ensure that there is no hoarding of RECs

c. Prohibition of traders and bilateral transactions in REC mechanism

d. None of the above

5) Consider the following statements with respect to Indian Elephants

1. It has been recognized as National Heritage Animal of India.

2. India has the largest population of Asian Elephants.

3. It has been given the highest degree of protection under the Indian Wildlife Protection Act (1972).

Which of the statements given above are correct?

a. 1 and 2 only

b. 1 and 3 only

c. 2 and 3 only

d. 1, 2 and 3

Answers

1. a

PM POSHAN Scheme

The Cabinet Committee on Economic Affairs (CCEA) has recently approved the continuation of ‗National Scheme for PM POSHAN in Schools‘ for the five year period 2021-22 to 2025-26.

The PM POSHAN Scheme for providing one hot cooked meal in Government and Government-aided schools.

This is a Centrally-Sponsored Scheme which covers all school children studying in Classes I-VIII of Government, Government-Aided Schools.

The earlier name of the scheme was ‗National Scheme for Mid Day Meal in Schools‘ popularly known as Mid Day Meal Scheme.

The scheme is proposed to be extended to students studying in pre-primary or Bal Vatikas of Government and Government-aided primary schools in addition to all the 11.80 crore children from elementary classes.

Social Audit of the scheme is made mandatory in all the districts.

2. a

SACRED Portal

The Ministry of Social Justice and Empowerment will launch the Senior Able Citizens for Re-Employment in Dignity (SACRED) Portal.

It is a first-of-its kind dedicated employment exchange for senior citizens seeking work opportunities.

Page 68: INDEX [iasparliament.s3.ap-south-1.amazonaws.com]

69

www.shankariasacademy.com | www.iasparliament.com

The portal will enable those above 60 years of age to search for jobs on the virtual platform.

The ministry, however, clarified that the portal does not guarantee jobs but will act as an enabler for those seeking employment.

Background

This portal is being seen as the way forward against the backdrop of a steady rise in the population of elderly.

The number of senior citizens has gone up from nearly 2 crore in 1951 to 7.6 crore in 2001 and about 10.4 crore in 2011.

As per the report of the technical group on population projections for India (2011-2036) submitted to National Commission on Population, the population estimates of senior citizens were 13.76 crore (10.1%) in 2021, which is predicted to rise to 16.28 crore (11.4%) by 2026 and go further up to 22.74 crore (14.9%) in 2036.

3. a

Trojan Asteroids

The Trojan asteroids are leftovers of the primordial material from which Jupiter and the other outer planets were formed.

The space rocks function similar to time capsules from the earliest days of the solar system, about 4.5 billion years ago.

The Trojans orbit the Sun in two loose groups, with one group leading ahead of Jupiter in its path, the other trailing behind.

These primitive bodies hold vital clues to deciphering the history of the solar system.

LUCY Misison

NASA‘s Lucy Mission will be the first space mission to study the Trojans.

The mission takes its name from the fossilized human ancestor (called ―Lucy‖ by her discoverers) whose skeleton provided unique insight into humanity's evolution.

Likewise, the Lucy mission will revolutionize our knowledge of planetary origins and the formation of the solar system.

4. c

Renewable Energy Certificate

Ministry of Power has redesigned the existing Renewable Energy Certificate (REC) mechanism, which was introduced in 2010.

The proposed changes will provide some flexibility to the players, additional avenues, rationalization and also addressing the RECs validity period uncertainty issues.

Features of changes proposed in revamped REC mechanism are,

1. Validity of REC would be perpetual i.e., till it is sold.

2. Floor and forbearance prices are not required to be specified.

3. Central Electricity Regulatory Commission (CERC) to have monitoring and the surveillance mechanism ensures that there is no hoarding of RECs.

4. RE generator who is eligible for REC, will be eligible for issuance of RECs for the period of PPA as per the prevailing guidelines.

The existing RE projects that are eligible for REC would continue to get RECs for 25 years.

5. Technology multiplier can be introduced to promote new and high priced RE technologies.

Issuance - RECs can be issued to obligated entities (including DISCOMs and open access consumers) which purchase RE Power beyond their RPO compliance notified by the Central Government.

No REC to be issued to the beneficiary of subsidies/concessions or waiver of any other charges. The FOR to define concessional charges uniformly for denying the RECs.

Allowing traders and bilateral transactions in REC mechanism.

The changes proposed in revamped REC mechanism will be implemented by CERC through regulatory process.

Page 69: INDEX [iasparliament.s3.ap-south-1.amazonaws.com]

70

www.shankariasacademy.com | www.iasparliament.com

5. d

Indian Elephant (Elephasmaximus)

It is a keystone species andan integral component of ecosystems playing a crucial role in maintaining forest ecosystem and biodiversity.

It has been recognized as National Heritage Animal of India and highest degree of protection has given under Indian Wildlife Protection Act (1972).

India has the largest population of Asian Elephants with 30000 wild and about 3600 captive elephants.